N54B Final

Réussis tes devoirs et examens dès maintenant avec Quizwiz!

Which of the following assessment findings would be expected in pulmonary embolism (PE)? (Select all that apply.) a. Chest pain b. Tachycardia c. Tachypnea d. Fever e. Hemoptysis

A, B, C, D, E Rationale: An individual with PE usually presents with the sudden onset of pleuritic chest pain, dyspnea, tachypnea, tachycardia, and unexplained anxiety. Occasionally syncope (fainting) or hemoptysis occurs. With large emboli, a pleural friction rub, pleural effusion, fever, and leukocytosis may be noted. Recurrent small emboli may not be detected until progressive incapacitation, precordial pain, anxiety, dyspnea, and right ventricular enlargement are exhibited. Massive occlusion causes severe pulmonary hypertension and shock.

A nurse is assessing a patient with hypoparathyroidism. Clinical manifestations of hypoparathyroidism include (select all that apply): a. Tetany b. Chvostek sign c. Trousseau sign d. Oily skin e. Hair loss

A, B, C, E Rationale: Symptoms of hypoparathyroidism includes tetany, Chvostek and Trousseau signs, dry (not oily) skin, and loss of body and scalp hair.

A nurse is teaching about urinary pathogens in men. Which information should the nurse include? Mechanisms for defense against urinary pathogens in men include (select all that apply): a. The long length of the urethra b. Alkaline pH of urine c. Secretion of mucus that traps bacteria d. Antimicrobial secretions from the prostate e. Implantation of the ureters in the bladder

A, D Rationale: Both the longer urethra and prostatic secretions decrease the risk of infection in men. The urine is not more alkaline, and they do not secrete mucus that traps bacteria. The ureters in men and women are implanted in similar positions and in their normal position; it is not a factor in the development of cystitis.

1. A patient has been researching cardiac cells on the internet. Which information indicates the patient has a good understanding? Cardiac cells can withstand ischemic conditions for _____ minutes before irreversible cell injury occurs.

ANS: 20

When the spouse of a patient experiencing a thrombotic stroke asks when "clot busting" treatment should begin, how should the nurse respond? Recommendations suggest that treatment for a thrombotic stroke begin within ________ hours of onset of symptoms.

ANS: 3

A 20-year-old male was recently diagnosed with lactose intolerance. He eats an ice cream cone and develops diarrhea. His diarrhea can be classified as _____ diarrhea. a. Osmotic b. Secretory c. Hypotonic d. Motility

ANS: A A nonabsorbable substance in the intestine leads to osmotic diarrhea. Infections lead to secretory diarrhea. Hypotonic diarrhea is not a form of diarrhea. Food is not mixed properly, digestion and absorption is impaired, and motility is increased leading to motility diarrhea.

A cause of hydrocephalus in subarachnoid hemorrhage is: a. Scarring of meninges and impairment of CSF resorption b. Choroid plexus injury c. Impairment of CSF flow through the ventricles d. Vasoconstriction related to carbon dioxide (CO2) changes in the cerebral circulation

ANS: A A primary cause of hydrocephalus is impairment of CSF resorption that allows fluid to accumulate.

A 32-year-old female presents with lower leg pain, with swelling and redness. While obtaining the patient's history, which finding could have caused her condition? a. Venous thrombus b. Heart valve damage c. Bacterial infection d. Atherosclerosis

ANS: A A thrombus formation in the vein leads to inflammation that may cause pain and redness with obstruction. Increased pressure in the vein behind the clot may lead to edema of the extremity.

A patient with an addiction to alcohol checked into a rehabilitation center. He experiences delirium, inability to concentrate, and is easily distracted. From which of the following is he most likely suffering? a. Acute confusional state b. Echolalia c. Dementia d. Dysphagia

ANS: A Delirium and the inability to concentrate are characteristics of acute confusional state. Echolalia is the repeating of words and phrases. Dementia is characterized by loss of recent and remote memory. Dysphagia is difficulty speaking.

When a student asks what can cause dilated, fixed pupils, what is the nurse's best response? Dilated fixed pupils can be caused by: a. Brainstem hypoxia b. Cerebral contusions c. Compression of the hypothalamus d. Spinal shock

ANS: A Dilated fixed pupils are caused by brainstem hypoxia.

A 30-year-old female presents to her primary care provider with fever, cardiac murmur, and petechial skin lesions. She is diagnosed with infective endocarditis. When the patient wants to know what caused the disease, what is the nurse's best response? The most likely cause of the disease is: a. Bacteria b. Viruses c. Fungi d. Parasites

ANS: A Infective carditis is due to a bacterial infection.

Which characteristic changes should the nurse keep in mind while caring for a patient with left heart failure? As left heart failure progresses: a. Left ventricular preload increases. b. Systemic vascular resistance decreases. c. Left end-diastolic volume decreases. d. Pulmonary vascular resistance decreases.

ANS: A Left ventricular preload increases in left heart failure because less blood is ejected from the left ventricle. Left heart failure does not lead to a decrease in systemic resistance; it leads to an increase in resistance. Left end-diastolic volume will increase. Pulmonary vascular resistance will increase.

A 45-year-old male presents with seizures. An MRI reveals a meningioma most likely originating from the: a. Dura mater and arachnoid membrane b. Astrocytes c. Pia mater d. CNS neurons

ANS: A Meningiomas tumors usually originate from the arachnoidal (meningeal) cap cells in the dura mater. Astrocytes are found in the brain, but are not related to meningiomas. The pia mater is the location of the infection meningitis. Neurons are located throughout all regions of the brain.

A nurse recalls the most common cardiac valve disease in the United States, which tends to be most prevalent in young women, is: a. Mitral valve prolapse b. Pulmonary stenosis c. Tricuspid valve prolapse d. Aortic stenosis

ANS: A Mitral valve prolapse is the most common valve disorder in the United States.

Most cases of combined systolic and diastolic hypertension have no known cause and are documented on the chart as _____ hypertension. a. Primary b. Secondary c. Congenital d. Acquired

ANS: A Most cases of hypertension are diagnosed as primary hypertension. Most cases of hypertension are diagnosed as primary hypertension, not secondary, which is due to a known cause.

A 56-year-old male presents to his primary care provider for a checkup. Physical exam reveals edema, hepatomegaly, and muffled heart sounds. Which of the following is of greatest concern to the nurse? a. Tamponade b. Exudate c. Aneurysm d. Pulsus paradoxus

ANS: A Muffled heart sounds is an indication of tamponade, and with tamponade the blood backs up into the venous system, leading to hepatomegaly.

A 20-year-old male is admitted to the neurological critical care unit with a severe closed head injury. When an intraventricular catheter is inserted, the ICP is recorded at 24 mm Hg. How should the nurse interpret this reading? This reading is: a. Higher than normal b. Lower than normal c. Normal d. Borderline

ANS: A Normal ICP is 5 to 15 mm Hg; at 24 mm Hg, the patient's is higher than normal.

Which of the following characteristics is associated with an acute occlusion of mesenteric blood flow to the small intestine? a. Often precipitated by an embolism b. Commonly associated with disease such as pancreatitis and gallstones c. Caused by chronic malnutrition and mucosal atrophy d. Often a complication of hypovolemic shock

ANS: A Occlusion of blood flow is often precipitated by embolism.

A patient has paralysis of both legs. What type of paralysis does the patient have? a. Paraplegia b. Quadriplegia c. Infraparaplegia d. Paresthesia

ANS: A Paraplegia is the paralysis of both legs. Quadriplegia is the paralysis of all four extremities. Infraparaplegia is not a description of paralysis. Paresthesia is a loss of sensation, not paralysis.

A 42-year-old female presents with abdominal discomfort, epigastric tenderness, and bleeding. Gastroscopy reveals degeneration of the gastric mucosa in the body and fundus of the stomach. Which of the following would most likely follow? a. Pernicious anemia b. Osmotic diarrhea c. Increased acid secretion d. Decreased gastrin secretion

ANS: A Pernicious anemia can develop because the damage to the mucosa makes the intrinsic factor less available to facilitate vitamin B12 absorption in the ileum.

A 50-year-old woman named Mrs. Jones came into the emergency room is complaining of fever, swelling in the feet, low urine volume, and red-tinged urine. An initial assessment finds her blood pressure to be 152/93. These symptoms indicate a potential diagnosis of: A. Acute nephritic syndrome B. Chronic nephritic syndrome C. Acute nephrotic syndrome D. Chronic nephrotic syndrome

ANS: A Rationale: edema, hematuria, low urine volume, and high blood pressure are all indicators of acute nephritic syndrome. They occur as a result of a decreased glomerular filtration rate and enhanced tubular reabsorption of salt and water. It is an acute inflammatory process that occludes the glomerular capillary lumen and damages the capillary wall

A 60-year-old male is diagnosed with cancer of the esophagus. Which of the following factors most likely contributed to his disease? a. Reflux esophagitis b. Intestinal parasites c. Ingestion of salty foods d. Frequent use of antacids

ANS: A Reflux is a factor in the development of esophageal cancer.

A 72-year-old female has a history of right heart failure caused by a right ventricular myocardial infarction. Which of the following symptoms are specifically related to her right heart failure? a. Significant edema to both lower legs and feet b. Hypertension c. Decreased urine output d. Dyspnea upon exertion

ANS: A Right-sided failure allows blood to back up into the systemic circulation, leading to peripheral edema.

A 65-year-old male is diagnosed with chronic pulmonary disease and elevated pulmonary vascular resistance. Which of the following heart failures should the nurse assess for in this patient? a. Right heart failure b. Left heart failure c. Low-output failure d. High-output failure

ANS: A Right-sided failure occurs when the patient experiences chronic pulmonary disease and elevated pulmonary vascular resistance because the blood has difficulty overcoming the pressure and blood builds up in the right side of the heart.

A 28-year-old female presents to the ER reporting severe chest pain that worsens with respirations or lying down. She has a fever, tachycardia, and a friction rub. Based upon the assessment findings, the nurse determines the patient is experiencing: a. Acute pericarditis b. Myocardial infarction (MI) c. Stable angina d. Pericardial effusion

ANS: A Severe chest pain that worsens with respirations or lying down in a patient with fever, tachycardia, and a friction rub is characteristic of acute pericarditis.

A 51-year-old male is at the health clinic for an annual physical exam. After walking from the car to the clinic, he developed substernal pain. He also reported discomfort in his left shoulder and his jaw, lasting 2 to 3 minutes and then subsiding with rest. He indicates that this has occurred frequently over the past few months with similar exertion. The nurse suspects he is most likely experiencing: a. Stable angina b. Unstable angina c. Prinzmetal angina d. Myocardial infarction (MI)

ANS: A Stable angina is associated with activity and subsides with rest. Unstable angina is a form of acute coronary syndrome that results from reversible myocardial ischemia and occurs at rest. Chest pain that occurs at rest and at night is descriptive of Prinzmetal angina. MI pain does not subside with rest.

A 50-year-old male visits the cardiologist for an EKG. Results indicate that he has no PR interval and a variable QRS rate with rhythm irregularity. Which of the following is the most likely diagnosis to be recorded on the chart? a. Atrial tachycardia b. Atrial fibrillation c. Sinus dysrhythmia d. Idioventricular rhythm

ANS: B Atrial fibrillation is characterized by no PR interval and a variable QRS response. Atrial tachycardia is characterized by a normal PR interval. Sinus dysrhythmia is characterized by a change in rhythm associated with inspiration and expiration. Idioventricular dysrhythmia is characterized by absent P waves.

A 42-year-old female is diagnosed with constrictive pericarditis. The nurse assesses the blood pressure for decreased cardiac output because of: a. Pericardial effusions b. Fibrosis and calcification of the pericardial layers c. Cardiomyopathy d. Hemorrhage in the pericardial cavity

ANS: B In constrictive pericarditis, fibrous scarring compresses the heart and eventually reduces cardiac output. Pericardial effusion is manifested by chest pain. Cardiomyopathy is a general term for pathophysiological changes in the heart. Hemorrhage in the pericardial cavity will lead to tamponade.

A 35-year-old male presents with pulmonary hypertension. Testing reveals he is in right heart failure. Which of the following is the most likely diagnosis the nurse will see listed on the chart? a. Aortic stenosis b. Tricuspid regurgitation c. Aortic regurgitation d. Mitral regurgitation

ANS: B Tricuspid regurgitation leads to volume overload in the right atrium and ventricle, increased systemic venous blood pressure, and right heart failure. Aortic stenosis is manifested by narrowed pulse pressure. Aortic regurgitation is manifested by widened pulse pressure resulting from increased stroke volume and diastolic backflow. Mitral regurgitation is manifested by heart failure, but not pulmonary hypertension.

A 68-year-old male presents to the ER reporting chest pain. He has a history of stable angina that now appears to be unstable. He most likely has: a.Mild to moderate atherosclerosis b. Impending myocardial infarction (MI) c. Electrical conduction problems in the heart d. Decreased myocardial oxygen demand

ANS: B Unstable angina is an indication of impending MI.

Which statement by the nurse indicates teaching was successful regarding CNS tumors in adults? The most common primary CNS tumors in adults are: a. Meningiomas b. Oligodendrogliomas c. Astrocytomas d. Ependymomas

ANS: C Astrocytomas are the most common primary tumors in the CNS, accounting for over 50%. Meningiomas tumors usually originate from the arachnoidal (meningeal) cap cells in the dura mater and account for 30% of tumors. Oligodendrogliomas account for about 2% of tumors. Ependymomas are more common in children.

A major contributing process in CVAs is the development of atheromatous plaques in cerebral circulation. These most commonly form: a. In the larger veins b. Near capillary sphincters c. In cerebral arteries d. In the venous sinuses

ANS: C Atheromatous plaques (stenotic lesion) form at branchings and curves in the cerebral circulation. Over 20 to 30 years, atheromatous plaques (stenotic lesion) form at branchings and curves in the cerebral circulation, primarily the arteries.

A 60-year-old male presents to his primary care provider reporting chest pain. He is diagnosed with atherosclerosis. This disease is caused by: a. Arterial wall thinning and weakening b. Abnormally dilated arteries and veins c. Abnormal thickening and hardening of vessel walls d. Autonomic nervous system imbalances

ANS: C Atherosclerosis is a form of arteriosclerosis characterized by thickening and hardening of the vessel wall.

A compensatory alteration in the diameter of cerebral blood vessels in response to increased intracranial pressure is called: a. Herniation b. Vasodilation c. Autoregulation d. Amyotrophy

ANS: C Autoregulation is the compensatory alteration in the diameter of the intracranial blood vessels designed to maintain a constant blood flow during changes in cerebral perfusion pressure. Herniation is the downward protrusion of the brainstem. Vasodilation is an enlargement in vessel diameter and a part of autoregulation, but the vessels should not dilate in the presence of increased intracranial pressure. Amyotrophy is involved with the anterior horn cells of the spinal cord and not related to autoregulation.

A 40-year-old male presents with epigastric pain. Tests reveal acute pancreatitis. The most likely cause of his condition is: a. Pancreatic duct obstruction by a malignant tumor b. Surgical trauma to the pancreas c. Obstruction of the biliary tract by a gallstone d. Toxic injury to the pancreas from nonprescription medications

ANS: C Biliary tract obstruction by gallstones is one of the known causes of pancreatitis.

Mr. Jones has been admitted to the ED with complications as a result of his unmanaged chronic renal failure. The nurse needs to transfer the patient. Because bone fractures are a risk factor in chronic renal failure, the nurse should recall that: A. Calcium is lost in the urine. B. Osteoblast activity is excessive. C. The kidneys fail to activate vitamin D. D. Autoantibodies to calcium molecules develop.

ANS: C Hypocalcemia is accelerated by impaired renal synthesis of 1,25-vitamin D. The combined effect of vitamin D deficiency can result in renal osteodystrophies with increased risk for fractures.

Manifestations associated with hepatic encephalopathy from chronic liver disease are the result of: a. Hyperbilirubinemia and jaundice b. Fluid and electrolyte imbalances c. Impaired ammonia metabolism d. Decreased cerebral blood flow

ANS: C Impaired ammonia metabolism leads to the symptoms of hepatic encephalopathy. Symptoms are primarily neurologic, not jaundice oriented.

A nurse is teaching staff about endocarditis. Which information should the nurse include? Inflammatory cells have difficulty limiting the colonization of microorganisms in infective endocarditis because the: a. Microorganisms are resistant. b. Valves are avascular. c. Microorganisms are sequestered in a fibrin clot. d. Colonies overwhelm the phagocytes.

ANS: C In endocarditis, bacterial colonies are inaccessible to host defenses because they are embedded in the protective fibrin clots.

When a person is in shock, a nurse remembers impairment in cellular metabolism is cause by: a. Release of toxic substances b. Free radical formation c. Inadequate tissue perfusion d. Lack of nervous or endocrine stimulation

ANS: C In shock, impaired cellular metabolism is caused by inadequate tissue perfusion.

A 33-year-old male is brought to the ER for treatment of injuries received in a motor vehicle accident. An MRI reveals an injury of the cervical cord. Cord swelling in this region may be life threatening because: a. Increased ICP may occur. b. Reflexes will be disrupted. c. Diaphragm function may be impaired. d. Bladder emptying will not occur.

ANS: C In the cervical region, cord swelling may be life threatening because it may impair the diaphragm function.

A 65-year-old female loses her balance while walking in the woods, causing her to fall and hit her head. She loses consciousness and is in a coma for 5 days. She is diagnosed as having diffuse brain injury. Which of the following would most likely occur in this patient? a. Complete loss of vision b. Arrhythmia c. Blunted affect d. Meningitis infection

ANS: C Individuals who experience diffuse brain injury experience agitation, impulsiveness, blunted affect, social withdrawal, and depression.

A patient has memory loss of events that occurred before a head injury. What cognitive disorder does the nurse suspect the patient is experiencing? a. Selective memory deficit b. Anterograde amnesia c. Retrograde amnesia d. Executive memory deficit

ANS: C Retrograde amnesia is manifested by loss of memory of past personal history memories or past factual memories. In selective memory deficit, the person reports inability to focus attention and has failure to perceive objects and other stimuli. Anterograde amnesia is a loss of the ability to form new memories. Executive memory deficit involves the failure to stay alert and oriented to stimuli.

A 10-year-old male presents with fever, lymphadenopathy, arthralgia, and nose bleeds. He is diagnosed with rheumatic heart disease. When a staff member asks what caused the disease, what is the nurse's most correct response? The most likely cause of this disease is: a. Congenital heart defects b. Human immunodeficiency virus (HIV) infections c. Group A ß-hemolytic streptococcus infections d. Acute pericarditis

ANS: C Rheumatic fever is a systemic, inflammatory disease caused by a delayed exaggerated immune response to infection by the group A â-hemolytic streptococcus.

A 50-year-old male with a 30-year history of smoking was diagnosed with bronchogenic cancer. He developed edema and venous distention in the upper extremities and face. Which of the following diagnosis will the nurse observe on the chart? a. Thromboembolism b. Deep vein thrombosis c. Superior vena cava syndrome (SVCS) d. Chronic venous insufficiency

ANS: C SVCS is a progressive occlusion of the superior vena cava that leads to venous distention in the upper extremities and head. Thromboembolism would not lead to the generalized symptoms described in the patient. Deep vein thrombosis would not lead to upper extremity symptoms. Chronic venous insufficiency would primarily affect one extremity.

A nurse is planning care for a patient in shock. Which principle should the nurse remember? During shock states, glucose uptake is usually: a. Enhanced b. Normal c. Impaired d. Energy intensive

ANS: C Some compensatory mechanisms activated by shock contribute to decreased glucose uptake by the cells.

A 50-year-old male presents with low back pain. He denies trauma and says he just woke up and it was hurting. An MRI reveals that the vertebra at L5 slid forward relative to those above and below it. Which of the following conditions will be documented on the chart? a. Degenerative disk disease b. Spondylolysis c. Spondylolisthesis d. Spinal stenosis

ANS: C Spondylolisthesis occurs when there is forward displacement of the deficient vertebra. Degenerative disk disease is a pathophysiological cause of spondylolisthesis, but is not the definition of forward displacement. Spondylolysis is a structural defect of the spine. Spinal stenosis is a narrowing of the spinal canal.

An 82-year-old female was admitted to the hospital with confusion and severe hypotension. Her body's compensatory mechanisms are increased heart rate, vasoconstriction, and movement of large volumes of interstitial fluid to the vascular compartment. What kind of shock does the nurse suspect the patient is experiencing? a. Anaphylactic b. Hypovolemic c. Neurogenic d. Septic

ANS: B In hypovolemic shock, heart rate and SVR increase, boosting both cardiac output and tissue perfusion pressures. Interstitial fluid moves into the vascular compartment. In anaphylactic shock, bronchoconstriction occurs with hypotension. In neurogenic shock, hypotension occurs, but fluid does not shift. In septic shock, interstitial fluid shift does not occur.

A 15-year-old male is brought to the ER for treatment of injuries received in a motor vehicle accident. An MRI reveals spinal cord injury, and his body temperature fluctuates markedly. The most accurate explanation of this phenomenon is that: a. He developed pneumonia. b. His sympathetic nervous system has been damaged and thermal control disturbed. c. He has a brain injury. d. He has septicemia from an unknown source.

ANS: B The patient experiences disturbed thermal control because the sympathetic nervous system is damaged. The hypothalamus cannot regulate body heat through vasoconstriction and increased metabolism; therefore, the individual assumes the temperature of the air.

A 52 year old women came in the ER stating she woke up with chills and a high fever. She has a constant ache on her lower back bilaterally. She states that she had difficulty urinating and she feels nauseous. Her vitals were, 101.8 degrees F, respiratory rate is 22 bpm, pulse is 86 bpm, BP is 120/80. When the nurse did a physical assessment she noted that palpation over the costovertebral angle causes pain bilaterally. What problem does this describe? A. Kidney stones B. Pyelonephritis C. Nephrotic syndrome D. Nephritic syndrome

ANS: B The patient presents CVA tenderness and has chills and high fever, this combined with dysuria indicates that it is pyelonephritis. Since the pain is constant it is not kidney stones.

A 75-year-old obese female presents to her primary care provider reporting edema in the lower extremities. Physical exam reveals that she has varicose veins. Upon performing the history, which of the following is a possible cause for the varicose veins? a. Extreme exercise b. Long periods of standing c. Trauma to the deep veins d. Ischemia

ANS: B The probable cause of the patient's varicose veins is gradual venous distention caused by the action of gravity on blood in the legs due to long periods of standing.

A 3-month-old female develops colicky pain, abdominal distention, and diarrhea after drinking cow's milk. The best explanation for her symptoms is: a. Deficiency of bile that stimulates digestive secretions and bowel motility b. Excess of amylase, which increases the breakdown of starch and causes an osmotic diarrhea c. Overgrowth of bacteria from undigested fat molecules, which leads to gas formation and decreased bowel motility d. Excess of undigested lactose in her digestive tract, resulting in increased fluid movement into the digestive lumen and increased bowel motility

ANS: D Undigested lactose increases the osmotic gradient in the intestine, causing irritation and osmotic diarrhea.

A 20-year-old male is brought to the emergency room (ER) for treatment of injuries received in a motor vehicle accident. A spinal cord injury is suspected. What two regions should the nurse assess as they are most likely to be damaged? a. Cervical and thoracic regions b. Thoracic and lumbar regions c. Lumbar and sacral regions d. Cervical and lumbar regions

ANS: D Vertebral injuries in adults occur most often at cervical and lumbar regions as these are the most mobile portions of the vertebral column.

A nurse is preparing to teach staff about the most common type of traumatic brain injury. Which type of traumatic brain injury should the nurse discuss? a. Penetrating trauma b. Diffuse axonal injury c. Focal brain injury d. Concussion

ANS: D A concussion is the most common type of traumatic brain injury. Penetrating trauma is due to such items as bullets and is not the most common type of brain injury. Diffuse axonal injury is due to rotation and is not as common as concussions. Focal brain injury is not as common as concussions.

Which of the following would increase a patient's risk for thrombotic stroke? a. Hyperthyroidism b. Hypertension c. Anemia d. Dehydration

ANS: D Dehydration is a risk factor because it increases blood viscosity and decreases cerebral perfusion. Hyperthyroidism would lead to increased blood pressure but does not place the patient at risk for thrombotic stroke. Hypotension, not hypertension is a risk factor for thrombotic stroke. Anemia would decrease a person's risk for thrombotic stroke.

A nurse thinks a patient may be experiencing dementia. Which assessment finding will most help support this diagnosis? a. Violent behavior b. Hyperactivity c. Depression d. Loss of recent and remote memory

ANS: D Dementia is characterized by loss of recent and remote memory.

_____ are most at risk of spinal cord injury from minor trauma. a. Infants b. Men c. Women d. The elderly

ANS: D Elderly people are particularly at risk from minor trauma that results in serious spinal cord injury because of preexisting degenerative vertebral disorders.

For an infection to progress to septic shock, which of the following factors should the nurse determine occurred? a. The individual must be immunosuppressed. b. The myocardium must be impaired. c. The infection must be gram negative. d. Bacteria must enter the bloodstream.

ANS: D For septic shock to occur, bacteria must enter the bloodstream.

A 30-year-old Caucasian female was recently diagnosed with primary hypertension. She reports that she eats fairly well, usually having red meat and potatoes daily. She also reports that her father has hypertension as well. A nurse determines which of the following risk factors is most likely associated with this diagnosis? a. Race b. Diet c. Age d. Genes

ANS: D Genetic factors, such as family history of hypertension, are the number one factor in the development of hypertension.

An adult has hydrocephalus. When the patient asks the nurse what caused this, how should the nurse respond? Hydrocephalus in adults is most often caused by: a. Overproduction of CSF b. Intercellular edema c. Elevated arterial blood pressure d. Defective CSF reabsorption

ANS: D Hydrocephalus occurs because of defective reabsorption of the fluid. Hydrocephalus can occur because of overproduction of CSF, but in adults it occurs most often because of defective reabsorption of the fluid.

A 52-year-old female presents with continuous abdominal pain that intensifies after eating. She is diagnosed with chronic pancreatitis. Contributing factors include: (Select all that apply.) a. Alcohol abuse b. Peptic ulcer disease c. Trauma d. Smoking e. Bulimia

B, C, D Pancreatitis can be acute or chronic, and risk factors include alcoholism, obstructive biliary tract disease (particularly cholelithiasis), peptic ulcers, trauma, hyperlipidemia, and smoking, as well as certain drugs.

Which of the following are characteristic of idiopathic pulmonary arterial hypertension (IPAH)? (Select all that apply.) a. Male gender b. Fatigue c. Dyspnea d. Jugular vein distention e. Weight gain

B, C, D Rationale: IPAH is a rare condition and usually occurs in women between the ages of 20 and 40. Manifestations of fatigue, chest discomfort, tachypnea, and dyspnea (particularly with exercise) are common. Examination may reveal peripheral edema, jugular venous distension, a precordial heave, and accentuation of the pulmonary component of the second heart sound.

A 26-year-old female recently underwent surgery and is now experiencing dyspnea, cough, fever, and leukocytosis. Tests reveal that she has a collapsed lung caused by removal of air from obstructed alveoli. What condition will the nurse observe on the chart? a. Compression atelectasis b. Bronchiectasis c. Absorption atelectasis d. Hypoventilation

c. Absorption atelectasis Rationale: Absorption atelectasis results from removal of air from obstructed or hypoventilated alveoli or from inhalation of concentrated oxygen or anesthetic agents. Compression atelectasis is caused by external pressure exerted by tumor, fluid, or air in pleural space or by abdominal distention pressing on a portion of lung. Bronchiectasis is dilation of the bronchi, not atelectasis. Hypoventilation is inadequate alveolar ventilation of the lungs; it is not due to removal of air.

A 30-year-old male is demonstrating hematuria with red blood cell casts and proteinuria exceeding 3 to 5 g/day, with albumin being the major protein. The most probable diagnosis the nurse will see documented on the chart is: a. Cystitis b. Chronic pyelonephritis c. Acute glomerulonephritis d. Renal calculi

c. Acute glomerulonephritis Rationale: Two major symptoms distinctive of more severe glomerulonephritis are: (1) hematuria with red blood cell casts and (2) proteinuria exceeding 3 to 5 g/day with albumin (macroalbuminuria) as the major protein.

A nurse checks lab results as both Cushing syndrome and Addison disease can manifest with elevated levels of: a. ADH b. Cortisol c. Adrenocorticotropic hormone (ACTH) d. Aldosterone

c. Adrenocorticotropic hormone (ACTH)

A 30-year-old male prison inmate contracted tuberculosis during an outbreak. While planning interactions, the nurse realizes the patient can transmit this disease through: a. Skin contact b. Fecal-oral contact c. Airborne droplets d. Blood transfusions

c. Airborne droplets

A 53-year-old male with a 20-year history of smoking is diagnosed with emphysema. When the nurse is asked what causes this, what is the nurse's best response? Changes in his lungs are caused by: a. Viral infections b. Destruction of alveolar macrophages c. Alpha-1-antitrypsin deficiency d. Fibrotic lung disease

c. Alpha-1-antitrypsin deficiency

When a patient asks what the most common type of renal stones is composed of, how should the nurse respond? The most common type of renal stone is composed of: a. magnesium. b. struvite. c. calcium. d. phosphate.

c. Calcium Rationale: The most common composition of a renal stone is calcium, accounting for 70% to 80%

Which of the following diseases should the nurse teach the patient to prevent as it is the ultimate cause of death in the patient with diabetes? a. Renal disease b. Stroke c. Cardiovascular disease d. Cancer

c. Cardiovascular disease

A 50-year-old male is diagnosed with pulmonary embolism (PE). Which of the following symptoms most likely occurred before treatment? a. Dry cough and inspiratory crackles b. Shallow respirations and wheezing c. Chest pain and shortness of breath d. Kussmaul respirations and back pain

c. Chest pain and shortness of breath Rationale: An individual with PE usually presents with the sudden onset of pleuritic chest pain, dyspnea, tachypnea, tachycardia, and unexplained anxiety.

A 25-year-old male presents with chronic bronchitis of 5 months' duration. When obtaining the patient's history, which of the following findings is most likely to cause this condition? a. Chronic asthma b. Air pollution c. Cigarette smoke d. Recurrent pneumonias

c. Cigarette smoke

A group of mountain climbers experience confusion, tachycardia, edema, and decreased renal output after climbing Mount Rainier. A nurse recalls this condition is caused by: a. Bronchoconstriction b. Hypoventilation c. Decreased inspired oxygen d. Diffusion abnormalities

c. Decreased inspired oxygen Rationale: The mountain climbers are experiencing decreased inspired oxygen. Bronchoconstriction would result in wheezing. Hypoventilation would result in retained CO2.

When the nurse observes a diagnosis of nosocomial pneumonia, the patient generally acquires this pneumonia: a. At day care centers b. On airplanes c. During hospitalization d. In the winter season

c. During hospitalization

Individuals with a recent diagnosis of emphysema should be assessed for which most common presenting factor? a. A productive cough b. Cyanosis c. Dyspnea d. Cor pulmonale

c. Dyspnea

What is the cause of the hyperpigmentation seen in people with Cushing syndrome? a. Abnormal levels of cortisol b. Permissive effects of aldosterone when cortisol levels are altered c. Elevated levels of ACTH d. Hypersensitivity of melanocytes with sun exposure

c. Elevated levels of ACTH Rationale: Bronze or brownish hyperpigmentation of the skin, mucous membranes, and hair occurs when there are very high levels of ACTH.

A nurse is preparing to teach the staff about asthma. Which information should the nurse include? Airway hyper-responsiveness in asthma is related to: a. Increased sympathetic nervous system response b. The release of stress hormones c. Exposure to an allergen causing mast cell degranulation d. Hereditary decrease in IgE responsiveness

c. Exposure to an allergen causing mast cell degranulation

A patient with Addison disease has weakness and easy fatigability. A nurse recalls this is due to: a. Hyperkalemia b. Hypoglycemia c. Hypocortisolism d. Metabolic acidosis

c. Hypocortisolism

A 13-year-old male who uses insulin to control his type 1 diabetes experiences hunger, lightheadedness, tachycardia, pallor, headache, and confusion during gym class. The most probable cause of these symptoms is: a. Hyperglycemia resulting from incorrect insulin administration b. Dawn phenomenon caused by eating a snack before gym class c. Hypoglycemia caused by increased exercise d. Somogyi effect caused by insulin sensitivity

c. Hypoglycemia caused by increased exercise

A 50-year-old female presents with lightheadedness and overall abnormal feelings. Hyperaldosteronism is diagnosed. Which of the following symptoms would the nurse expect? a. Hypovolemia b. Hypotension c. Hypokalemia d. Hyponatremia

c. Hypokalemia Rationale: Hypokalemia occurs due to increased renal secretion of potassium. Hypervolemia, not hypovolemia, occurs. Hypertension, not hypotension, occurs. Hypernatremia, not hyponatremia, occurs

A 45-year-old female with Graves disease underwent surgical removal of her thyroid gland. During the postoperative period, her serum calcium was low. The most probable reason for her low serum calcium is: a. Hyperparathyroidism secondary to Graves disease b. Myxedema secondary to surgery c. Hypoparathyroidism caused by surgical injury to the parathyroid glands d. Hypothyroidism resulting from lack of thyroid replacement

c. Hypoparathyroidism caused by surgical injury to the parathyroid glands

A nurse is reviewing the results of an ABG and finds reduced oxygenation of arterial blood. What term should the nurse use to describe this condition? a. Ischemia b. Hypoxia c. Hypoxemia d. Hypocapnia

c. Hypoxemia Rationale: Hypoxemia is a reduction of oxygen in arterial blood. Ischemia is a lack of blood supply to tissues. Hypoxia is reduced oxygen in tissues. Hypocapnia is decreased CO2.

A 12-year-old female is newly diagnosed with type 1 DM. When the parents ask what causes this, what is the nurse's best response? a. A familial, autosomal dominant gene defect b. Obesity and lack of exercise c. Immune destruction of the pancreas d. Hyperglycemia from eating too many sweets

c. Immune destruction of the pancreas Rationale: The most common cause of type 1 DM is a slowly progressive autoimmune T cell-mediated disease that destroys the beta cells of the pancreas.

A 42-year-old male is involved in a motor vehicle accident during which he loses a lot of blood. The nurse realizes he is in acute renal failure caused by: a. Kidney stones b. Immune complex deposition in the glomerulus c. Inadequate renal blood flow d. Obstruction of the proximal tubule

c. Inadequate renal blood flow Rationale: With blood loss, renal failure is due to inadequate blood flow. Kidney stones will lead to postrenal renal failure. Intrarenal renal failure is due to glomerular nephritis. Blood loss would not cause obstruction of the proximal tubule.

A 50-year-old male patient presents with polyuria and extreme thirst. He was given exogenous ADH. For which of the following conditions would this treatment be effective? a. Neurogenic diabetes insipidus b. Psychogenic diabetes insipidus c. Nephrogenic diabetes insipidus d. SIADH

a. Neurogenic diabetes insipidus Rationale: Neurogenic diabetes insipidus is caused by the insufficient secretion of ADH; thus, exogenous ADH would be useful in the treatment of this disorder.

A 65-year-old male recently had a cerebrovascular accident that resulted in dysphagia. He now has aspiration of gastric contents. The nurse assesses the patient for which complication? a. Pneumonia b. Bronchiectasis c. Pneumothorax d. Emphysema

a. Pneumonia

An 80-year-old female is in the hospital for a bone fracture. While there she develops a large, nonlethal pulmonary embolus. Which of the following is a direct result of the obstruction to pulmonary blood flow? a. Pulmonary hypertension b. Systemic hypertension c. Pulmonary edema d. Risk of cerebral emboli

a. Pulmonary hypertension Rationale: Significant obstruction of the pulmonary vasculature leads to increased pulmonary artery pressures (pulmonary hypertension). The pressure is in the pulmonary bed; it is not systemic. Pulmonary edema occurs secondary to heart failure. Pulmonary emboli obstruct blood flow in the pulmonary vasculature, they do not increase the risk of cerebral emboli.

A 27-year-old male has a severe kidney obstruction leading to removal of the affected kidney. Which of the following would the nurse expect to occur? a. Atrophy of the remaining kidney b. Compensatory hypertrophy of the remaining kidney c. Dysplasia in the remaining kidney d. Renal failure

b. Compensatory hypertrophy of the remaining kidney Rationale: The remaining kidney would hypertrophy to compensate for the increased workload of the loss of the affected kidney.

A 20-year-old male presents to his primary care provider reporting difficulty breathing when lying down. What term should the nurse use to document this condition? a. Dyspnea b. Orthopnea c. Apnea d. Tachypnea

b. Orthopnea Rationale: Orthopnea is dyspnea that occurs when an individual lies flat. Dyspnea is shortness of breath that occurs with activity. Apnea is cessation of breathing. Tachypnea is rapid breathing.

A nurse assesses a patient with a complicated urinary tract infection (UTI) for: a. Several species of bacteria b. Other health problems c. Urosepsis d. Urethral obstruction

b. Other health problems Rationale: A complicated UTI develops when there is an abnormality in the urinary system or a health problem that compromises host defenses or response to treatment. UTI may occur alone or in association with pyelonephritis, prostatitis, or kidney stones.

A 47-year-old male is diagnosed with pulmonary edema. Which assessment findings will the nurse observe? a. Thick mucus secretions b. Pink, frothy sputum c. Hypocapnia d. Wheezing

b. Pink, frothy sputum

An 11-year-old male is newly diagnosed with type 1 DM. Which classic symptoms should the nurse assess the patient for? a. Recurrent infections, visual changes, fatigue, and paresthesias b. Polydipsia, polyuria, polyphagia, and weight loss c. Vomiting; abdominal pain; sweet, fruity breath; dehydration; and Kussmaul breathing d. Weakness, vomiting, hypotension, and mental confusion

b. Polydipsia, polyuria, polyphagia, and weight loss

A 60-year-old male undergoes surgery for a bone fracture. Which of the following nursing measures would be most effective for preventing pulmonary embolism (PE) in this patient? a. Ensure that patient uses supplemental oxygen. b. Prevent deep vein thrombosis formation. c. Check hematocrit and hemoglobin levels frequently during the postoperative period. d. Promote aggressive fluid intake.

b. Prevent deep vein thrombosis formation. Rationale: PE most commonly results from embolization of a clot from deep venous thrombosis (DVT) involving the lower leg; thus preventing these will help prevent pulmonary emboli.

A 25-year-old male was diagnosed with Goodpasture syndrome. While planning care for this patient, which of the following mechanisms would cause tissue injury? a. Viral infection of the Bowman capsule b. Production of antibodies against the glomerular basement membrane c. Antigen-antibody complex deposition with complement activation d. Abnormal activation of clotting factors and microclotting in the glomerulus

b. Production of antibodies against the glomerular basement membrane

A 28-year-old female presents with fever, chills, and flank pain. She is diagnosed with pyelonephritis. A nurse recalls the patient's infection is located in the: a. Bladder b. Renal pelvis c. Renal tubules d. Glomerulus

b. Renal pelvis Rationale: Pyelonephritis is an infection of one or both upper urinary tracts (ureter, renal pelvis, and interstitium).

A 22-year-old female has a low level of TSH. What condition does the nurse expect the patient is experiencing? a. Primary hypothyroidism b. Secondary hypothyroidism c. Autoimmune hypothyroidism d. Atypical hypothyroidism

b. Secondary hypothyroidism Rationale: Causes of secondary hypothyroidism are related to either pituitary or hypothalamic failure, which would be evident by low levels of TSH. Primary hypothyroidism would be evident by elevated levels of TSH. Autoimmune hypothyroidism would be evident by elevated TSH. Atypical hypothyroidism would be evident by normal or elevated TSH.

A nurse remembers a low ventilation-perfusion ratio results in: a. Increased dead space b. Shunting c. Alveolar collapse d. Bronchoconstriction

b. Shunting Rationale: Hypoxemia caused by inadequate ventilation of well-perfused areas of the lung is a form of mismatching called shunting.

A 54-year-old male is diagnosed with empyema. Upon receiving and reviewing the culture result, which organism does the nurse suspect is the most likely cause? a. Virus b. Staphylococcus aureus c. Klebsiella pneumonia d. Moraxella catarrhalis

b. Staphylococcus aureus

A 75-year-old male reports to his primary care provider loss of urine with cough, sneezing, or laughing. Which of the following is the most likely diagnosis the nurse will observe on the chart? a.Urge incontinence b. Stress incontinence c. Overflow incontinence d. Functional incontinence

b. Stress incontinence

A 50-year-old male presents with hypotension, hypoxemia, and tracheal deviation to the left. Tests reveal that the air pressure in the pleural cavity exceeds barometric pressure in the atmosphere. Based upon these assessment findings, what does the nurse suspect the patient is experiencing? a. Pleural effusion b. Tension pneumothorax c. Open pneumothorax d. Transudative pneumothorax

b. Tension pneumothorax Rationale: Tracheal deviation suggests tension pneumothorax.

When a nurse is assessing the physical features of individuals with Cushing syndrome, these findings will include: a. Weight loss and muscle wasting b. Truncal obesity and moon face c. Pallor and swollen tongue d. Depigmented skin and eyelid lag

b. Truncal obesity and moon face Rationale: Weight gain is the most common feature and results from the accumulation of adipose tissue in the trunk, facial, and cervical areas. These characteristic patterns of fat deposition have been described as "truncal obesity," "moon face," and "buffalo hump."

What common neurologic disturbances should the nurse assess for in a patient with a pituitary adenoma? a. Coma b. Visual disturbances c. Confused states d. Breathing abnormalities

b. Visual disturbances Rationale: The clinical manifestations of pituitary adenomas are visual changes including visual field impairments (often beginning in one eye and progressing to the other) and temporary blindness.

Which patient would the nurse assess for paroxysmal nocturnal dyspnea (PND)? A patient with: a. Pulmonary fibrosis b. Asthma c. Left ventricular failure d. Hypotension

c. Left ventricular failure Rationale: Some individuals with cardiac disease awake at night gasping for air and have to sit up or stand to relieve the dyspnea (PND). PND occurs primarily with cardiac disease, not pulmonary fibrosis. Individuals with asthma experience orthopnea, rather than PND.

A nurse wants to determine if there is kidney dysfunction in a patient with diabetes. Which of the following is the earliest manifestation? a. Polyuria b. Glycosuria c. Microalbuminuria d. Decreased glomerular filtration

c. Microalbuminuria

A nurse is preparing to teach the staff about asthma. Which information should the nurse include? Airway obstruction contributing to increased airflow resistance and hypoventilation in asthma is caused by: a. Type II alveolar cell injury and decreased surfactant b. Alveolar fibrosis and pulmonary edema c. Mucus secretion, bronchoconstriction, and airway edema d. Collapse of the cartilaginous rings in the bronchi

c. Mucus secretion, bronchoconstriction, and airway edema Rationale: The mediators of asthma cause vasodilation, increased capillary permeability, mucosal edema, bronchial smooth muscle contraction (bronchospasm), and mucus secretion from mucosal goblet cells with narrowing of the airways and obstruction to airflow.

A nurse is caring for a patient with SIADH. What severe complication should the nurse assess for? a. Stroke b. Diabetes insipidus c. Neurologic damage d. Renal failure

c. Neurologic damage Rationale: When the hyponatremia of SIADH becomes severe, 110 mEq/L to 115 mEq/L, confusion, lethargy, muscle twitching, convulsions, and severe and sometimes irreversible neurologic damage may occur.

A 30-year-old male was diagnosed with thyroid carcinoma. The lab tests the nurse would most likely find are _____ T3 and T4 levels. a. High b. Low c. Normal d. Variable

c. Normal Rationale: Most individuals with thyroid carcinoma have normal T3 and T4 levels and are therefore euthyroid.

When a patient wants to know what most commonly causes hypoparathyroidism, how should the nurse reply? It is most commonly caused by: a. Pituitary hyposecretion b. Parathyroid adenoma c. Parathyroid gland injury d. Hypothalamic inactivity

c. Parathyroid gland injury

A 42-year-old female is diagnosed with chronic renal failure, and the nurse is discussing dietary treatment. Which information indicates the nurse understands dietary regimen? Treatment includes restricting: a. Fats b. Complex carbohydrates c. Proteins d. Sugars

c. Proteins Rationale: Low-protein diets are recommended.

A 25-year-old male presents to his primary care provider reporting changes in facial features. CT scan reveals a mass on the anterior pituitary, and lab tests reveal severely elevated growth hormone (GH). Which of the following would the nurse also expect to find? a. Decreased IGF-1 b. Hypotension c. Sexual dysfunction d. Height increases

c. Sexual dysfunction

A 35-year-old female with Graves disease is admitted to a medical-surgical unit. Which of the following symptoms would the nurse expect to find before treatment? a. Weight gain, cold intolerance b. Slow heart rate, rash c. Skin hot and moist, rapid heart rate d. Constipation, confusion

c. Skin hot and moist, rapid heart rate Rationale: Symptoms of Graves disease include heat intolerance and increased tissue sensitivity to stimulation by the sympathetic division of the autonomic nervous system. Weight loss, rather than weight gain, and heat intolerance would result. Tachycardia, not slow heart rate, would occur. Diarrhea would occur as opposed to constipation.

A 10-year-old male is brought to the ER with prolonged bronchospasm and severe hypoxemia. The most likely diagnosis on the chart is: a. Exercise-induced asthma b. Chronic obstructive pulmonary disease (COPD) c. Status asthmaticus d. Bronchiectasis

c. Status asthmaticus Rationale: When bronchospasm is not reversed by usual measures, the individual is considered to have severe bronchospasm or status asthmaticus.

While turning a patient with chronic renal failure, which principle should the nurse recall? Bone fractures are a risk factor in chronic renal failure because: a. Calcium is lost in the urine. b. Osteoblast activity is excessive. c. The kidneys fail to activate vitamin D. d. Autoantibodies to calcium molecules develop.

c. The kidneys fail to activate vitamin D. Rationale: Hypocalcemia is accelerated by impaired renal synthesis of 1,25-vitamin D. The combined effect of vitamin D deficiency can result in renal osteodystrophies with increased risk for fractures.

While planning care for a patient who has acute pyelonephritis. A nurse recalls the most common condition associated with the development of acute pyelonephritis is: a. Cystitis b. Renal cancer c. Urinary tract obstruction d. Nephrotic syndrome

c. Urinary tract obstruction Rationale: Urinary obstruction and reflux of urine from the bladder are the most common underlying risk factors.

A 25-year-old female is diagnosed with urinary tract obstruction. While planning care, the nurse realizes that the patient is expected to have hydronephrosis and a decreased glomerular filtration rate caused by: a. decreased renal blood flow. b. decreased peritubular capillary pressure. c. dilation of the renal pelvis and calyces proximal to a blockage. d. stimulation of antidiuretic hormone.

c. dilation of the renal pelvis and calyces proximal to a blockage.

A 15-year-old male was diagnosed with pharyngitis. Eight days later he developed acute glomerulonephritis. While reviewing the culture results, which of the following is the most likely cause of this disease? a. Klebsiella b. Human immunodeficiency virus (HIV) c. Genital herpes virus d. Group A ß-hemolytic streptococcus

d. Group A ß-hemolytic streptococcus

A 28-year-old male reports to his primary care provider that he has had a cold for a week and is coughing up bloody secretions. When giving report, what term should the nurse use to describe this condition? a. Hematemesis b. Cyanosis c. Rhinitis d. Hemoptysis

d. Hemoptysis Rationale: Hemoptysis is the coughing up of bloody secretions. Hematemesis is bloody vomiting. Cyanosis is a bluish color to the skin. Rhinitis is a runny nose.

Which of the following alterations would the nurse expect to find in a patient with untreated Cushing disease or syndrome? a. Bradycardia b. Tachypnea c. Hyperkalemia d. Hypertension

d. Hypertension Rationale: With elevated cortisol levels, vascular sensitivity to catecholamines increases significantly, leading to vasoconstriction and hypertension. Tachycardia is more likely than bradycardia due to increased sensitivity to catecholamines. Tachypnea does not occur; the patient experiences hypertension. Hypokalemia, not hyperkalemia, occurs.

A 25-year-old male presents with fatigue, constipation, and sexual dysfunction. Tests reveal all pituitary hormones are normal and no masses are present. The nurse suspects the most likely cause of his symptoms is a dysfunction in the: a. Anterior pituitary b. Posterior pituitary c. Pars intermedia d. Pituitary stalk

d. Pituitary stalk Rationale: When pituitary hormones are normal, dysfunction in the action of hypothalamic hormones are most commonly related to interruption of the connection between the hypothalamus and pituitary, the pituitary stalk.

A 56-year-old male presents with flank pain and polyuria. Tests reveal that he has an enlarged prostate. Which of the following types of renal failure should the nurse monitor for as it is the most likely to occur? a. Prerenal b. Intrarenal c. Extrarenal d. Postrenal

d. Postrenal Rationale: The patient will experience postrenal renal failure due to obstruction by the prostate.

A 54-year-old female is diagnosed with nephrotic syndrome. Which of the following is a common symptom of this disease? a. Hematuria b. Dysuria c. Oliguria d. Proteinuria

d. Proteinuria Rationale: Nephrotic syndrome is manifested by proteinuria. Nephrotic syndrome is diagnosed when the protein level in a 24-hour urine collection is greater than 3.5 g.

A 25-year-old male presents with chronic bronchitis of 5 months' duration. Which of the following is the most significant concern for the nurse to monitor in this patient? a. Left heart failure b. Pulmonary embolus c. Immunosuppression d. Recurrent pulmonary infections

d. Recurrent pulmonary infections

When a patient has a massive pulmonary embolism (PE), what complications will the nurse monitor for? a. Damage to the lung microcapillaries b. Chronic obstructive pulmonary disease (COPD) c. Disseminated intravascular coagulation (DIC) d. Shock and death

d. Shock and death

Diabetes insipidus, diabetes mellitus (DM), and SIADH share which of the following assessment manifestations? a. Polyuria b. Edema c. Vomiting and abdominal cramping d. Thirst

d. Thirst

A 60-year-old female with a history of cirrhosis presents with dyspnea, impaired ventilation, and pleural pain. A diagnosis of pleural effusion is made, and a watery fluid is drained. When giving report, the nurse will refer to this fluid as: a. Exudative b. Purulent c. Infected d. Transudative

d. Transudative Rationale: Pleural effusions that enter the pleural space from intact blood vessels can be transudative (watery).

A 35-year-old female took corticosteroid therapy for several months. Which of the following would the nurse expect to find? a. Renal toxicity b. Episodes of hypoglycemia c. Hypotension d. Type 2 DM

d. Type 2 DM Rationale: Overt DM develops in approximately 20% of individuals with hypercortisolism. Hypertension, not hypotension, occurs; hypotension occurs with Addison.

A patient asks what causes pneumonia. How should the nurse reply? Pneumonia is caused by: a. Use of anesthetic agents in surgery b. Atelectasis c. Chronic lung changes seen with aging d. Viral or bacterial infections

d. Viral or bacterial infections

A 24-year-old female is diagnosed with renal calculus that is causing obstruction. Which of the following symptoms would she most likely experience? a. Anuria b. Hematuria c. Pyuria d. Flank pain

d. Flank pain

A 73-year-old female has increased pulmonary pressure resulting in right heart failure. The nurse should monitor for a possible complication because a potential cause for her heart to fail is: a. Hypertension b. Left heart failure c. Acute pneumonia d. Pericarditis

ANS: B Right-sided failure often follows left-sided failure when pulmonary congestion forces backward flow of blood into the left ventricle.

While planning care for a patient with superior vena cava syndrome (SVCS), which principle should the nurse remember? SVCS is a progressive _____ of the superior vena cava (SVC) that leads to venous distention of the upper extremities and head. a. Inflammation b. Occlusion c. Distention d. Sclerosis

ANS: B SVCS is a progressive occlusion of the SVC that leads to venous distention in the upper extremities and head.

Pancreatic insufficiency is manifested by deficient production of: a. Insulin b. Amylase c. Lipase d. Bile

ANS: C Pancreatic insufficiency is the deficient production of lipase by the pancreas.

Bob who is a 56-year-old male was recently entered the hospital with many complications. Bob has been diagnosed with type 2 diabetes and has high blood pressure. He has smoked for over 20 years and continues to smoke 2 packs a day. Bob also has a medical history of drinking 4 or more beers daily. During the nurse's physical assessment on Bob, she noted he is complaining of lower back pain and has not been using the bathroom as often as usual despite still intaking the same amount of liquids. He also feels nauseous, very tired all the time, has gained 4 pounds in the last 48 hours, and sometimes has shortness of breath. His blood results show he has had an increase in creatinine levels which is at 2 mg. From this data collections what would the nurse diagnose Bob with. A. Acute Kidney Injury B. Renal Dysfunction C. Urinary tract Obstruction D. Kidney Stones

ANS: A According the acute kidney injury definition in the book, it can be defined as decline and inability to have a high renal output. The patient was experiencing all symptoms related to acute kidney injury from low urinary output, fatigue, SOB, it all happened in less than 48 hours, and lastly, his creatinine levels increased from normal range which definitely makes it acute kidney injury.

A 39-year-old female with chronic intermittent pain in the epigastric area 2 to 3 hours after eating is diagnosed with a duodenal ulcer. Which of the following behaviors may have contributed to the development of the ulcer? a. Cigarette smoking b. Drinking caffeinated beverages c. Consuming limited fiber d. Antacid consumption

ANS: A Acid production is stimulated by cigarette smoking.

In alcoholic cirrhosis, hepatocellular damage is caused by: a. Acetaldehyde accumulation b. Bile toxicity c. Acidosis d. Fatty infiltrations

ANS: A Alcoholic cirrhosis is caused by the toxic effects of alcohol metabolism on the liver. Alcohol is transformed to acetaldehyde, and excessive amounts significantly alter hepatocyte function and activate hepatic stellate cells, a primary cell involved in liver fibrosis.

While planning care for a patient with an extradural hematoma, which principle should the nurse remember? The main source of bleeding in extradural (epidural) hematomas is: a. Arterial b. Venous c. Capillary d. Sinus

ANS: A An artery is the source of bleeding in 85% of extradural hematomas.

A nurse notes that a patient walks with the leg extended and held stiff, causing a scraping over the floor surface. What type of gait is the patient experiencing? a. Spastic gait b. Cerebellar gait c. Basal ganglion gait d. Scissors gait

ANS: A An individual who walks with the leg extended and held stiff, causing a scraping over the floor surface, is experiencing a spastic gait. A cerebellar gait is wide-based with the feet apart and often turned outward or inward for greater stability. A basal ganglion gait occurs when the person walks with small steps and a decreased arm swing. A scissors gait is associated with bilateral injury and spasticity. The legs are abducted so they touch each other.

One consequence of switching from aerobic to anaerobic cellular metabolism during shock states is: a. Decreased adenosine triphosphate (ATP) production b. Cellular dehydration c. Cellular alkalosis d. Free radical formation

ANS: A Anaerobic metabolism leads to decreased ATP production.

A 60-year-old female with a history of alcoholism complains of recent weight gain and right flank pain. Physical examination reveals severe ascites. This condition is caused by decreased: a. Albumin and lack of cellular integrity b. Capillary filtration pressure c. Capillary permeability d. Antidiuretic hormone secretion

ANS: A Ascites is due to decreased albumin and lack of cellular integrity.

An attorney spoke to the nurses regarding brain death. Which statement indicates the nurses understood brain death? For legal purposes, brain death is defined as: a. Cessation of brain function b. Lack of cortical function c. A VS d. Death of the brainstem

ANS: A Brain death occurs when there is cessation of function of the entire brain, including the brainstem and cerebellum. Lack of cortical function is not enough to define brain death. A VS is complete unawareness of the self or surrounding environment and complete loss of cognitive function. Brain death occurs when there is cessation of function of the entire brain, including the brainstem and cerebellum, not just the brainstem.

The patient is experiencing an increase in intracranial pressure. This increase results in: a. Brain tissue hypoxia b. Intracranial hypotension c. Ventricular swelling d. Expansion of the cranial vault

ANS: A Brain tissue hypoxia occurs as a result of increased intracranial pressure as it places pressure on the brain. Increased intracranial pressure leads to intracranial hypertension. Ventricular swelling may lead to increased intracranial pressure, but increased pressure does not lead to ventricular swelling. Expansion of the cranial vault may lead to increased intracranial pressure, but increased pressure does not lead to expansion of the vault.

The breathing pattern that reflects respirations based primarily on carbon dioxide (CO2) levels in the blood is: a. Cheyne-Stokes b. Ataxic c. Central neurogenic d. Normal

ANS: A Cheyne-Stokes respirations occur as a result of CO2 levels in the blood. Ataxic breathing occurs as a result of dysfunction of the medullary neurons. Central neurogenic patterns occur as a result of uncal herniation. Normal respirations are based on the levels of oxygen (O2) in the blood.

Clinical manifestations of bile salt deficiencies are related to poor absorption of: a. Fats and fat-soluble vitamins b. Water-soluble vitamins c. Proteins d. Minerals

ANS: A Clinical manifestations of bile salt deficiency are related to poor intestinal absorption of fat and fat-soluble vitamins (A, D, E, K).

The cardinal signs of small bowel obstruction are: a. Vomiting and distention b. Diarrhea and excessive thirst c. Dehydration and epigastric pain d. Abdominal pain and rectal bleeding

ANS: A Colicky pains followed by vomiting and distention are the cardinal symptoms of small bowel obstruction.

A 25-year-old male was in an automobile accident. At impact, his forehead struck the windshield. In this situation, a nurse recalls the coup injury would occur in the _____ region. a. Frontal b. Temporal c. Parietal d. Occipital

ANS: A Coup injuries move forward and affect the frontal region of the brain. Coup injuries affect the frontal region of the brain as the brain moves forward, not to the side or temporal region. Contrecoup injuries affect the rear or occipital region.

Patients with myasthenia gravis often have tumors or pathologic changes in the: a. Brain b. Pancreas c. Thymus d. Lungs

ANS: C Patients with myasthenia gravis experience tumors in the thymus.

Mrs. Damian is a 64-year-old Filipino woman who came in her primary doctor's office. She complains that whenever she laughs, coughs, or sneeze she involuntarily wets herself. What type of incontinence does Mrs. Damian have? A. Stress B.Urge C. Overflow D. UTI

ANS: A Stress incontinence is characterized by involuntary loss of urine when the intra- abdominal pressure increases, and the pressure is not equally distributed in the urethra. Changes in the intra-abdominal pressure includes coughing, laughing or sneezing. This kind of incontinence happens when the intravesical pressure exceeds the maximum urethral closure pressure. It is common in all ages of women due to the weakness or disruption of the pelvic floor muscles leasing to an altered support of the vesicourethral sphincters

Susie, a 40-year-old white female patient has chronic kidney disease (CKD). She experiences pruritus, vomiting, and fatigue. In addition, she has a metallic taste in her mouth and a plus two edema. Her respiratory rate is 29 per minute and her glomerular filtration rate is 15 mL/min. Which of the following is Susie most likely experiencing: A. Metabolic Acidosis B. Metabolic Alkalosis C. Respiratory Acidosis D. Respiratory Alkalosis

ANS: A Susie's symptoms show she has an abnormal acid-base balance. Chronic kidney disease and vomiting usually lead to metabolic acidosis. Her glomerular filtration rate indicates she has stage 4 chronic kidney disease, which means her kidneys are not able to remove acid from the blood, leading to acid buildup (metabolic acidosis).

Mr. Adams, age 80, is experiencing frequent urination, hesitancy, a need to strain to initiate urination, a weak and small stream, and termination of the stream before the bladder is completely emptied. He is found to have benign prostate hypertrophy (BPH) and therefore is diagnosed with a lower urinary tract obstruction. What will be the bladders first compensatory mechanism? a. The bladder begins to hypertrophy and the wall becomes more sensitive to stretch stimulus. b. The bladder releases urine to release the pressure. c. The inner bladder surface forms smooth folds and is replaced by trabeculae. d. The bladder has no available compensation and is fatigued.

ANS: A The first thing that will happen is the bladder will hypertrophy in order to accomodate for the increasing pressure from the urine that is not able to be released.

Of the following groups, who are at highest risk for a cerebrovascular accident (CVA)? a. Blacks over 65 years of age b. Whites over 65 years of age c. Blacks under 65 years of age d. Whites under 65 years of age

ANS: A The individuals at highest risk for cerebrovascular accident would be blacks over 65. Blacks are at greater risks than whites. Older adults are at greater risk than younger adults.

A patient reports acute, intermittent, and excruciating pain in the upper outer quadrant of abdomen. They are brought to the ER and is constantly moving. The nurse notices that they have cool and clammy skin and are nauseous and vomiting. These clinical manifestations and a nuclear scintigraphy diagnostic help find that the patient has one of the major causes of UPPER urinary obstruction known as ...? A. Nephrolithiasis/kidney stones B. Renal Cell Carcinoma C. Pyelonephritis D. IgA nephropathy

ANS: A The major manifestation of nephrolithiasis is pain. This is the colicky pain that accompanies the collecting system or ureter. They usually present with acute, intermittent excruciating pain. Most often in the upper outer abdominal quadrant that may later radiate to other areas such as the lower abdomen, bladder, perineum, and scrotum for men.

A neurologist is teaching about brain injuries. Which information should the neurologist include? The most severe diffuse brain injury caused by rotational acceleration is most likely to be located _____ the brainstem. a. More peripheral to b. In the central portion of c. Throughout d. Distal to

ANS: A The most severe axonal injuries are located more peripheral to the brainstem.

A 56 year old female went to the Emergency Department complaining of a sudden onset of abdominal pain, nausea, vomiting, polyuria, and flank pain. Upon examination, the patient also had positive CVA signs and a recent UTI, what is wrong with the patient? A. Acute Pyelonephritis B. Renal Calculi (Kidney Stones) C. Polycystic Kidney Disease D. Renal Cysts

ANS: A The patients usually have had a previous urinary tract infection. This results from a gram negative bacterial infection usually caused by Escherichia coli, Proteus, Klebsiella, Enterobacter, and Pseudomonas. The bacteria ascend from the bladder to the kidneys. It can also spread through the bloodstream, which is less common. The onset is sudden with chill, back or groin pain, nausea, vomiting, polyuria, pyuria, and malaise. The costovertebral angle can be percussed with pain on the infected side.

A 68-year-old female is experiencing left heart failure. Physical exam reveals elevated blood pressure. The nurse understands this is most likely caused by: a. Sympathetic nervous system compensation for decreased cardiac output b. Stress hormones promoting increased cardiac contractility c. Cardiotoxic effects of catecholamines and angiotensin d. Diastolic dysfunction

ANS: A The sympathetic nervous system increases peripheral vascular resistance (PVR) and leads to hypertension.

A 15-month-old child from Pennsylvania was brought to the ER for fever, seizure activity, cranial palsies, and paralysis. Which of the following diagnosis will be documented in the chart? a. Eastern equine encephalitis b. Venezuelan encephalitis c. St. Louis encephalitis d. West Nile encephalitis

ANS: A The symptoms indicate encephalitis, and given the residence of the child and the symptoms, the diagnosis is Eastern equine encephalitis. Venezuelan occurs in Texas, Florida, and the South. St. Louis occurs in Canada and the Pacific coast. West Nile occurs throughout the United States, but primarily affects the elderly.

A 50-year-old male complains of frequently recurring abdominal pain, diarrhea, and bloody stools. A possible diagnosis would be: a. Ulcerative colitis b. Hiatal hernia c. Pyloric obstruction d. Achalasia

ANS: A Ulcerative colitis is manifested by fever, elevated pulse rate, frequent diarrhea (10 to 20 stools/day), urgency, obviously bloody stools, and continuous lesions present in the colon. Hiatal hernia is most often asymptomatic and would not be manifested by abdominal pain. Pyloric obstruction would be manifested by forceful or projectile vomiting. Achalasia would be manifested by difficulty or uncomfortable swallowing.

A patient comes in and reports back and abdominal pain, reports that her urine is dark and foul smelling. After the nurse assesses she reports that the bladder is inflamed.These clinical manifestations describe: A. Kidney Stones B. Cystitis C. Acute pylenephritis D. Hydronephrosis

ANS: B Most individuals with cystitis report symptoms of abdominal or back pain, dark concentrated urine, dysuria, and foul smelling urine. As far as the assessment part, people with cystitis usually have inflammation in the bladder.

A 20-year-old woman, with no history of urinary tract abnormalities or previous medical conditions, states that she has been feeling shaking chills, a constant ache in the loin area of the back, and a general feeling of malaise. Upon checking her vitals, she has a moderate fever. These clinical manifestations describe: A. Uncomplicated acute pyelonephritis B. Chronic pyelonephritis C. Complicated acute pyelonephritis D. Cystitis

ANS: A Uncomplicated acute pyelonephritis frequently occurs in young healthy women without structural or urinary tract obstructions or other causative factors. The onset of acute pyelonephritis is usually abrupt, with shaking chills, moderate to high fever, and a constant ache in the loin area of the back that can be unilateral or bilateral.

A 40-year-old female presents complaining of pain near the midline in the epigastrium. Assuming the pain is caused by a stimulus acting on an abdominal organ, the pain felt is classified as: a. Visceral b. Somatic c. Parietal d. Referred

ANS: A Visceral pain arises from a stimulus (distention, inflammation, ischemia) acting on an abdominal organ. Somatic is a form of parietal pain. Parietal pain, from the parietal peritoneum, is more localized and intense than visceral pain, which arises from the organs themselves. Referred pain is visceral pain felt at some distance from a diseased or affected organ.

A nurse recalls acute orthostatic hypotension can be caused by (select all that apply): a. Prolonged immobility b. Drug action c. Starvation d. Volume depletion e. Exercise

ANS: A, B, C, D Acute orthostatic hypotension occurs as a result of drug action, prolonged immobility, starvation, and volume depletion. Physical exhaustion, rather than exercise, could cause orthostatic hypotension.

A 31-year-old female presents with midabdominal pain. She is expected to have acute pancreatitis. Which of the will be part of the treatment plan? (Select all that apply.) a. Narcotic analgesics b. Restriction of food intake c. Nasogastric suctioning d. Antibiotics e. IV fluids

ANS: A, B, C, D, E Narcotic medications may be needed to relieve pain. To decrease pancreatic secretions and "rest the gland," oral food and fluids may be withheld, and continuous gastric suction is instituted. Nasogastric suction may not be necessary with mild pancreatitis, but it helps to relieve pain and prevent paralytic ileus in individuals who are nauseated and vomiting. Parenteral fluids are essential to restore blood volume and prevent hypotension and shock. Antibiotics may control infection. The risk of mortality increases significantly with the development of infection or pulmonary, cardiac, and renal complications.

A 22-year-old male underwent brain surgery to remove a tumor. Following surgery, he experienced a peptic ulcer. His ulcer is referred to as a(n) _____ ulcer. a. Infectious b. Cushing c. Ischemic d. Curling

ANS: B A Cushing ulcer is a stress ulcer associated with severe head trauma or brain surgery that results from decreased mucosal blood flow and hypersecretion of acid caused by overstimulation of the vagal nerve.

Mr. Lee is an overall healthy 75-year-old patient who was admitted to the hospital due to severe suprapubic pain. X-ray showed that he is suffering from benign prostatic hyperplasia. What urinary symptom a nurse can expect and what interventions can the nurse implement? A. Increased urinary output; suggest Mr. Lee to carry out Kegel exercises to increase control over frequent urination. B. Decreased urinary output; insert a urinary catheter to help Mr. Lee void. C. Small and weak stream; suggest Mr. Lee to carry out Kegel exercises. D. Formation of kidney stones; insert a urinary catheter to help Mr. Lee void.

ANS: B A benign prostatic hyperplasia commonly signifies an enlarged prostate gland. With an enlarged prostate gland, the urethra is suppressed and become obstructed, thus, resulting in a decreased urinary output. The bladder will compensate by undergoing hypertrophy with hopes to help the body release the accumulated urine. Simultaneously, the trigone area and intraureteric ridge of the bladder will also undergo hypertrophy resulting in a backpressure on the ureters, leading to the eventual development of hydroureters, causing kidney damage. If urine remains in the kidney, a urinary tract infection would result. Thus, to prevent this from happening, a proactive nurse will be quick to insert a catheter to help Mr. Lee void his urine.

A 75-year-old male experienced a lacunar stroke. When looking through the history of the patient's chart, which of the following would the nurse expect to find? a. An embolus b. Hypertension c. A hemorrhage d. An aneurysm

ANS: B A lacunar stroke is associated with hypertension.

A patient is admitted with amyotrophic lateral sclerosis (ALS). Which classic assessment findings will support the diagnosis of ALS? a. Progressive dementia b. Muscle weakness and atrophy c. Severe paresthesias d. Autonomic dysfunctions

ANS: B ALS is manifested by muscle weakness and atrophy.

A 35-year-old female arrives to the clinic complaining of hematuria and oliguria. She has also noticed that she has gained five pounds in the past week. Her past history is significant for systemic lupus erythematosus (SLE). Her blood pressure is 145/95 (she has no history of hypertension), and the rest of her vital signs are within normal limits. A urinalysis (UA) is sent and shows elevated red blood cells and protein. Which diagnosis best explains this patient's symptoms? A. Primary nephritic syndrome B. Secondary nephritic syndrome C. Primary nephrotic syndrome D. Secondary nephrotic syndrome

ANS: B Acute nephritic syndrome is characterized by a sudden onset of hematuria, proteinuria, decreased GFR, oliguria, signs of impaired renal function, edema, and hypertension. Her rapid weight gain and newly onset hypertension is likely due to decreased renal function. This is more likely to be a secondary condition, as the patient has a history of SLE.

A 65-year-old male recently suffered a cerebral vascular accident. He is now unable to recognize and identify objects by touch because of injury to the sensory cortex. How should the nurse document this finding? a. Hypomimesis b. Agnosia c. Dysphasia d. Echolalia

ANS: B Agnosia is the failure to recognize the form and nature of objects. Hypomimesis is a disorder of communication. Dysphasia is an impairment of comprehension of language. Echolalia is the ability to repeat.

A 10-year-old female was brought to the ER following a sudden onset of convulsions. The primary care provider thinks that she experienced an explosive, disorderly discharge of cerebral neurons referred to as: a. Reflex b. Seizure c. Inattentiveness d. Brain death

ANS: B An explosive disorderly discharge of neurons is a seizure. A reflex is an expected response. Inattentiveness is a form of neglect. Brain death is a cessation of function.

An infant is diagnosed with noncommunicating hydrocephalus. What is an immediate priority concern for this patient? a. Metabolic edema b. Interstitial edema c. Vasogenic edema d. Ischemic edema

ANS: B An immediate concern for the infant with noncommunicating hydrocephalus is interstitial edema.

A 15-year-old male who is allergic to peanuts eats a peanut butter cup. He then goes into anaphylactic shock. Which assessment findings will the nurse assess for? a. Bradycardia, decreased arterial pressure, and oliguria b. Bronchoconstriction, hives or edema, and hypotension c. Hypertension, anxiety, and tachycardia d. Fever, hypotension, and erythematous rash

ANS: B Anaphylactic shock is characterized by bronchoconstriction, hives, and hypotension.

Which information indicates the nurse understands assessment findings of an aneurysm? a. A headache is the most common symptom b. The majority are asymptomatic c. Nosebleeds are an early symptom d. Epidural hemorrhage occurs in over 80% of patients

ANS: B Aneurysms often are asymptomatic.

A 51-year-old male presents with recurrent chest pain on exertion. He is diagnosed with angina pectoris. When he asks what causes the pain, how should the nurse respond? The pain occurs when: a. Cardiac output has fallen below normal levels. b. The myocardial oxygen supply has fallen below demand. c. Myocardial stretch has exceeded the upper limits. d. The vagus nerve is stimulated.

ANS: B Angina is chest pain caused by myocardial ischemia, which develops if the flow or oxygen content of coronary blood is insufficient to meet the metabolic demands of myocardial cells. A decrease in cardiac output would lead to general systemic symptoms, not just chest pain, which is due to a decrease in myocardial oxygenation. Myocardial stretch does not affect angina symptoms. When the vagus nerve is stimulated, a change in rate occurs; it does not precipitate chest pain.

A 67-year-old female was previously diagnosed with rheumatic heart disease. Tests reveal lipoprotein deposition with chronic inflammation that impairs blood flow from the left ventricle into the aorta. Which of the following is the most likely diagnosis recorded on the chart? a. Aortic regurgitation b. Aortic stenosis c. Mitral regurgitation d. Mitral stenosis

ANS: B Aortic stenosis would impair blood flow from the left ventricle to the aorta. Aortic regurgitation would allow blood to flow back into the left ventricle. Mitral regurgitation would allow blood to flow from the left ventricle to the left atrium. Mitral stenosis would impair blood flow from the left atrium to the left ventricle.

A patient wants to know what causes atherosclerosis. How should the nurse respond? In general, atherosclerosis is caused by: a. High serum cholesterol levels b. Endothelial injury and inflammation c. An increase in antithrombotic substances d. Congenital heart disease

ANS: B Atherosclerosis begins with injury to the endothelial cells that line artery walls. High serum cholesterol levels are associated with atherosclerosis, but are not its cause.

A 25-year-old female presents to her primary care provider reporting fever, headache, nuchal rigidity, and decreased consciousness. She was previously treated for sinusitis. Which of the following is the most likely diagnosis? a. Aseptic meningitis b. Bacterial meningitis c. Fungal meningitis d. Nonpurulent meningitis

ANS: B Bacterial meningitis can occur secondary to sinusitis and is manifested by fever, tachycardia, chills, and a petechial rash with a severe throbbing headache, severe photophobia, and nuchal rigidity. The clinical manifestations of aseptic meningitis are milder than bacterial meningitis and are not associated with a previous infection such as sinusitis. Fungal meningitis presents as dementia. Nonpurulent meningitis is the same as aseptic and is milder and not associated with conditions such as sinusitis.

A 13-year-old female confides to her mother that she binge eats and induces vomiting to prevent weight gain. This disease is referred to as: a. Anorexia nervosa b. Bulimia nervosa c. Long-term starvation d. Laxative abuse

ANS: B Binge eating and vomiting is characteristic of bulimia nervosa. Anorexia nervosa is starvation eating.

Harmony, a three year old, was admitted to the pediatric unit at Ronald Regan Hospital with a tentative diagnosis of nephrotic syndrome. Once it has been confirmed, which unexpected finding would the nurse report? a. Proteinuria b. Blood in Urine c. Distended Abdomen d. Elevated Serum Lipid Levels

ANS: B Blood in the urine is rare in nephrotic syndrome but it appears in those with acute glomerulonephritis. Nephrotic Syndrome correlates with the proliferative inflammation of the glomerular capillary lumen and it damages the capillary wall. It is characterized by the sudden onset of hematuria but often times is a more advanced diagnosis

Cholecystitis is inflammation of the gallbladder wall usually caused by: a. Accumulation of bile in the hepatic duct b. Obstruction of the cystic duct by a gallstone c. Accumulation of fat in the wall of the gallbladder d. Viral infection of the gallbladder

ANS: B Cholecystitis can be acute or chronic, but both forms are almost always caused by a gallstone lodged in the cystic duct.

Ms. Smith, a 48 year old African-American woman, is admitted to the hospital complaining of nausea, pruritus, difficulty sleeping, and dependent edema. She has had hypertension for the past 10 years and has been out of medications for some years now. Ms smith also has a history of diabetes. Her BP is 180/100, pulse of 75, respirations of 22, BUN levels of 160, a GFR of 60 mil/min/1.73m^2for longer than 3 months, and a positive urine dipstick test (1+). What is the most likely diagnosis for Ms. Smith? A. Right-sided heart failure B. Chronic kidney disease C. Acute kidney injury D. Hypocalcemia

ANS: B Chronic Kidney Disease (CKD) is the significant loss in renal function over a long period of time and affects nearly all organ systems. It results from a number of conditions such as diabetes, hypertension, glomerulonephritis, polycystic kidney disease, and systemic lupus erythematosus. Common clinical manifestations are nausea, pruritus, difficulty sleeping, lower extremity edema, changes in her urinary habits, alterations in fluid & electrolyte and acid-base balance, etc. Since CKD is characterized as a destruction of nephrons, filtration and reabsorption is disrupted leading to an increase in urea(hence increased BUN levels) , creatinine (indicated with GFR), and albumin (urine dipstick test).

Chronic gastritis is classified according to the: a. Severity b. Location of lesions c. Patient's age d. Signs and symptoms

ANS: B Chronic gastritis is classified as type A (fundal) or type B (antral), depending on the pathogenesis and location of the lesions.

A 69-year-old male with a history of alcohol abuse presents to the emergency room (ER) after a month-long episode of headaches and confusion. Based on his alcoholism, a likely cause of his neurologic symptoms is: a. Concussion b. Chronic subdural hematoma c. Epidural hematoma d. Subacute subdural hematoma

ANS: B Chronic subdural hematomas are commonly found in persons who abuse alcohol and develop over weeks to months. A concussion is more acute, and chronic subdural hematomas are more commonly found in persons who abuse alcohol. It is a chronic subdural hematoma that occurs in alcoholics, not an epidural. Subacute subdural hematomas present with confusion, but these occur sooner than the chronic subdural associated with alcoholism.

Which of the following findings in the patient with Raynaud disease would indicate a need for further teaching? a. The patient eats bananas twice a day. b. The patient smokes cigarettes. c. The patient wears mittens outside. d. The patient takes calcium channel blockers.

ANS: B Cigarette smoking should be stopped to eliminate the vasoconstricting effects of nicotine.

10-month old Ryan was admitted into the PICU due to CKD related to an unknown cause. Which of these conditions is NOT likely to be the cause of his diagnosis? A. Glomerulonephritis B. Congestive Heart Failure C. Polycystic Kidney Disease D. Hyperoxaluria

ANS: B Congestive heart failure is not a cause of infantile CKD; it is a dominant manifestation indicating acute glomerulonephritis in older adults

When a patient asks how bad the injuries will be from a spinal injury, what is the nurse's best response? It is difficult to know the full extent of the injury because of: a. Incomplete transection of the spinal cord b. Swelling within the spinal cord c. Necrosis of the spinal cord d. Free radical injury and scarring of the glial cells

ANS: B Cord swelling increases the individual's degree of dysfunction, so it is hard to distinguish functions permanently lost from those temporarily impaired.

A 52-year-old female is diagnosed with coronary artery disease. The nurse assesses for myocardial: a. Hypertrophy b. Ischemia c. Necrosis d. Inflammation

ANS: B Coronary artery disease leads to myocardial ischemia.

A patient is admitted into the ER with burning and pain upon urination. He is concerned he is peeing too much, and his urine is cloudy and smelly. What is the most likely cause of his pain: A. Kidney stones B. Cystitis C. Bladder cancer D. Bladder dysfunction caused by spinal cord injury

ANS: B Cystitis (bladder infection) is characterized by lower abdominal or back discomfort, frequency of urination, and burning and pain upon urination. Often the urine becomes cloudy and smelly.

Individuals with Raynaud disease need to be counseled to avoid which of the following conditions to prevent severe symptoms? a. Allergic reactions b. Cold exposure c. Hot water immersion d. Tissue injury

ANS: B Raynaud disease consists of vasospastic attacks triggered by brief exposure to cold.

A 24 y.o. Asian female comes into the clinic for an antidepressant medication refill. As the provider is exiting the room at the end of the appointment, the woman stops her and asks, "Oh, I meant to mention this, but I've been having some issues in the last day or two... I've been needing to pee really often and it's kind of painful." The physician sits back down and asks some follow-up questions. The patient continues, "Yeah, my lower back has been hurting quite a bit and it burns when I pee. Sometimes the color of my pee is kind of cloudy and it smells worse than usual. I've been really embarrassed so I haven't told anybody except my mom." Her vitals are as follows: BMI: 20, temperature: 98.6, respirations: 20, heart rate: 75, BP: 110/70. Looking back in the medical record to see recent laboratory tests, serum values show a 4.1% HbA1c level. Patient appears to be in good health. Based on this information, what can the provider conclude is happening? A. Diabetes mellitus B. Cystitis C. Kidney stone D. Acute pyelonephritis

ANS: B Cystitis, also known as a bladder infection, manifests with frequent urination, pain in the lower back or abdomen, and painful, burning voiding. The urine may be cloudy or foul smelling, but other signs of infection such as a fever are not normally present in adults. The patient is most likely experiencing a bladder infection, which will likely resolve within 2 days of treatment

A 34 year-old patient, Mr. Johnson, developed acute tubular necrosis after undergoing major surgery. He is currently in the recovery phase, a period in which renal tissue begins to repair. During this phase, there is a gradual increase in urine output and a decrease in serum creatinine levels. What does these manifestations indicate? A. renal blood flow has been restored B. the nephrons have started to recover C. the urethra is no longer obstructed D. the bladder has started to recover

ANS: B During the recovery phase of acute tubular necrosis. a gradual increase in urine output and a decrease in serum creatinine levels indicate that the nephrons have recovered enough for urine excretion.

An increase in the rate of red blood cell breakdown causes which form of jaundice? a. Obstructive b. Hemolytic c. Hepatocellular d. Metabolic

ANS: B Excessive hemolysis (breakdown) of red blood cells can cause hemolytic jaundice (prehepatic jaundice).

The risk of hypovolemic shock is high with acute mesenteric arterial insufficiency because: a. The resulting liver failure causes a deficit of plasma proteins and a loss of oncotic pressure. b. Ischemia alters mucosal membrane permeability, and fluid is shifted to the bowel wall and peritoneum. c. Massive bleeding occurs in the GI tract. d. Overstimulation of the parasympathetic nervous system results in ischemic injury to the intestinal wall.

ANS: B Fluid shifts lead to hypovolemia.

The hypotensive state seen in septic shock can lead to: a. Bradycardia due to an overworked heart b. Gut lining disruption and the translocation of bacteria into the bloodstream c. Decreased oxygen demand of highly metabolic tissue d. The increased spread of infection

ANS: B Gastrointestinal mucosa changes cause the translocation of bacteria from the gut into the bloodstream. Tachycardia, rather than bradycardia, occurs. The hypotensive state would lead to an increased demand of all tissues. The hypotensive state would decrease the spread of infection.

A 16-year-old male fell off the bed of a pickup truck and hit his forehead on the road. He now has resistance to passive movement that varies proportionally with the force applied. He is most likely suffering from: a. Spasticity b. Gegenhalten c. Rigidity d. Dystonia

ANS: B Gegenhalten is manifested by resistance to passive movement that varies in direct proportion to force applied. Spasticity is manifested by a gradual increase in tone causing increased resistance until tone suddenly reduces. Rigidity is manifested by muscle resistance to passive movement of a rigid limb that is uniform in both flexion and extension throughout the motion. Dystonia is manifested by sustained involuntary twisting movement.

A 15-year-old male suffered diffuse brain injury after wrecking an all-terrain vehicle. He had momentary confusion and retrograde amnesia after 5 to 10 minutes. His injury could be categorized as: a. Grade I b. Grade II c. Grade III d. A mild concussion

ANS: B Grade II is manifested as momentary confusion and retrograde amnesia after 5 to 10 minutes. Grade I is manifested by confusion and disorientation with momentary amnesia. Grade III is manifested by confusion and retrograde amnesia from impact—also anterograde amnesia. A mild concussion does not result in memory loss.

A nurse in a nursing care facility is providing a lesson to her patients about a lower urinary tract infection: cystitis. She informs the patients that a cystitis is an infection characterized by frequency of urination, lower abdominal or back dis- comfort, and burning and pain on urination (Page, 1096). She also gave them a lesson about the risk factors for cystitis. Which of her patient's response best implies that they have a good understanding of risk factors for cystitis? A. A 75-year-old male patient says "since I am a male, I have a very low risk for cystitis" B. A 38 years old cancer patient receiving chemotherapy says "even though I am only 38, I am at risk for cystitis because of my weak immune system" C. A 17 years old sexually active female says "I guess I don't have to worry being at risk because I am only 17 and not at risk for cystitis since I have a strong immune system" D. Both B and C

ANS: B Healthy males under 50 years of age are at a significantly lower risk for lower urinary tract infections than women of the same age. The reason is because males have a longer urethra which makes it harder for the bacteria to ascend that far and infect the bladder. Additionally, prostatic fluids contain antimicrobial properties, further protecting them against cystitis and other lower urinary tract infections. Even though the male cancer patient is only 38, lack of a stronger immune system to fight off an infection puts him at risk

A 40-year-old male complains of uncontrolled excessive movement and progressive dysfunction of intellectual and thought processes. He is experiencing movement problems that begin in the face and arms that eventually affect the entire body. The most likely diagnosis is: a. Tardive dyskinesia b. Huntington disease c. Hypokinesia d. Alzheimer disease

ANS: B Huntington disease is manifested by chorea, abnormal movement that begins in the face and arms, eventually affecting the entire body. There is progressive dysfunction of intellectual and thought processes. Tardive dyskinesia is manifested by rapid, repetitive, stereotypic movements. Most characteristic is continual chewing with intermittent protrusions of the tongue, lip smacking, and facial grimacing. Hypokinesia is loss of voluntary movement despite preserved consciousness and normal peripheral nerve and muscle function. Alzheimer disease is manifested by cognitive deficits and not movement problems; motor impairments will occur in the later stages.

A 75-year old man by the name of Al Rene walks into the clinic and reports that he has been experiencing urinary frequency at night. Despite the need to use the restroom, he states that his stream has been weaker and small. He also states that he's been feeling a bit embarrassed because he's been experiencing dribbling, and incontinence despite using the bathroom often. Urinalysis demonstrates that his urine is light yellow, and clear. The only procedure he's had is a transurethral resection for benign prostatic hyperplasia a few months ago. Given this information, what is most likely the reason for Mr. Rene's complications? A. Nephrotic Syndrome B. Lower Tract Obstruction C. Kidney Stones D. Cystitis

ANS: B In lower tract obstructions, the kidneys produce urine normally, but the bladder does not fully expel it. In males, the most common cause is prostate gland enlargement which produces external compression of the urethra. As this compression progresses, symptoms of urinary frequency, strain to initiate urination, termination of the urine stream prior to the bladder completely emptying, a weak stream, hesitancy, and nocturia occurs. Unaddressed, the bladder becomes overstretched, and eventually loses its ability to contract. This can lead to overflow incontinence. Additionally, men who receive transurethral resection for BPH can experience urine leakage due to the bladder pressure exceeding the closure pressure at the urethral outlet - this is known as stress incontinence. This is the only option that presents with normal urine.

A 10-year-old male was climbing on a house and fell. He suffered a severe brain injury. His Glasgow Coma Scale (GCS) was 5 initially and 7 after 1 day. He remained unconscious for 2 weeks, then was confused and suffered from anterograde amnesia. Which of the following is he most likely experiencing? a. Mild diffuse brain injury b. Moderate diffuse brain injury c. Severe diffuse brain injury d. Postconcussive syndrome

ANS: B In moderate diffuse axonal injury, the score on the GCS is 4 to 8 initially and 6 to 8 by 24 hours and the person is confused and suffers a long period of posttraumatic anterograde and retrograde amnesia. Individuals who experience diffuse brain injury experience agitation, impulsiveness, blunted affect, social withdrawal, and depression. In severe diffuse axonal injury, the person experiences immediate autonomic dysfunction that disappears in a few weeks. Increased ICP appears 4 to 6 days after the injury. In postconcussive syndrome the individual experiences headache, nervousness or anxiety, irritability, insomnia, depression, inability to concentrate, forgetfulness, and fatigability.

A 49-year-old male presents to his primary care provider reporting chest pain. EKG reveals ST elevation. He is diagnosed with myocardial ischemia. Which of the following interventions would be most beneficial? a. Administer a diuretic to decrease volume. b. Apply oxygen to increase myocardial oxygen supply. c. Encourage exercise to increase heart rate. d. Give an antibiotic to decrease infection.

ANS: B Increase myocardial oxygen supply is indicated to treat ischemia. Oxygen is indicated, not a decrease in volume. Heart rate should be decreased to decrease cardiac workload. Antibiotics are not the most beneficial; oxygen is.

A 39-year-old male suffers a severe brain injury when he falls off a building while working. CT scan reveals that he has a basilar skull fracture. Based upon his injuries, what major complication should the nurse observe for in this patient? a. Hematoma formation b. Meningeal infection c. Increased intracranial pressure (ICP) d. Cognitive deficits

ANS: B Individuals with basilar skull fractures should be observed for meningitis and other complications.

A male patient complains of tiring easily, has difficulty rising from a sitting position, and cannot stand on his toes. The nurse would expect a diagnosis of: a. Parkinson disease b. Hypotonia c. Huntington disease d. Paresis

ANS: B Individuals with hypotonia tire easily (asthenia) or are weak. They may have difficulty rising from a sitting position, sitting down without using arm support, and walking up and down stairs, as well as an inability to stand on their toes. Individuals with Parkinson disease have rigidity and stiffness. Symptoms of Huntington disease include irregular, uncontrolled, and excessive movement. Paresis, or weakness, is partial paralysis with incomplete loss of muscle power.

A 34-year-old male was diagnosed with a bacterial GI infection. Which of the following types of diarrhea would most likely occur with his condition? a. Osmotic b. Secretory c. Hypotonic d. Motility

ANS: B Infections lead to secretory diarrhea. A nonabsorbable substance in the intestine leads to osmotic diarrhea. Hypotonic diarrhea is not a form of diarrhea. Food is not mixed properly, digestion and absorption is impaired, and motility is increased, leading to motility diarrhea.

Which of the following conditions is thought to contribute to the development of obesity? a. Insulin excess b. Leptin resistance c. Adipocyte failure d. Malabsorption

ANS: B Leptin resistance disrupts hypothalamic satiety signaling and promotes overeating and excessive weight gain and is a factor in the development of obesity. Insulin becomes resistant, not present in excess. Leptin resistance, not adipocyte failure, leads to obesity. Malabsorption does not lead to obesity, but primarily to weight loss.

When thought content and arousal level are intact but a patient cannot communicate, the patient has: a. Cerebral death b. Locked-in syndrome c. Dysphagia d. Cerebellar motor syndrome

ANS: B Locked-in syndrome occurs when the individual cannot communicate through speech or body movement but is fully conscious, with intact cognitive function. In cerebral death the person is in a coma with eyes closed. Dysphagia is difficulty speaking Cerebellar motor syndrome is characterized by problems with coordinated movement.

A 50-year-old male was admitted to the intensive care unit with a diagnosis of acute myocardial infarction (MI). He is being treated for shock. His cardiopulmonary symptoms include low blood pressure, tachycardia, and tachypnea. His skin is pale and cool. The primary cause of his shock is most likely: a. Rapid heart rate b. Decreased cardiac contractility c. Increased capillary permeability d. Decreased afterload due to vasodilation

ANS: B MI leads to decreased cardiac contractility due to a damaged myocardium and would lead to shock.

A nurse monitors the patient for _____ when rapid onset of malignant hypertension results. a. Atherosclerosis b. Encephalopathy c. Pulmonary edema d. Acute renal failure

ANS: B Malignant hypertension leads to cerebral edema and cerebral dysfunction (encephalopathy) and even death.

A neurologist is teaching about encephalitis. Which information should the neurologist include? Most causes of encephalitis are: a. Bacterial b. Viral c. Fungal d. Toxoid

ANS: B Most causes of encephalitis are viral.

Mr. Young, a healthy 18 year old Asian-American man visits his primary care provider due to a chief complaint of abnormal patterns of urination for the past 4 days. Following the health interview and physical exam, the provider believes the patient has a urinary tract infection because the patient is sexually active and an avid swimmer. To confirm this diagnosis, the provider orders a urine test which yields negative results for a urinary tract infection. A week later, Mr. Young returns, only this time his symptoms have worsened. He now states that his urine is dark, foamy, and it is even more painful to urinate than before. He also mentions that he has been had a lack of appetite, has been feeling fatigued, and has felt abnormally bloated. Additionally, he states that his feet feel swollen. The provider orders a more advanced urine analysis, which comes back positive for proteinuria. Which of the following conditions is the patient most likely experiencing and what is most likely the cause of his condition? A. Diagnosis: IgA Nephropathy, Cause: Viral Infection B. Diagnosis: Nephrotic Syndrome, Cause: Idiopathic C. Diagnosis: Nephrotic Syndrome, Cause: Diabetes Mellitus D. Diagnosis: IgA Nephropathy, Cause: Idiopathic

ANS: B Nephrotic Syndrome is characterized by excessive protein in the urine (≥ 3.5 g/day in adult), lipiduria, hypoalbuminemia, generalized edema, and hyperlipidemia. Nephrotic syndrome is not a specific disease but rather a series of clinical findings that occur due to glomerular damage. When there is damage to the glomerular membrane, it causes there to be increased permeability which leads to proteins escaping into the urine, and excessive loss of albumin. Additionally, a generalized edema can occur due to the decrease in plasma colloidal osmotic pressure, which results from loss of albumin in the vascular compartment. The edema is first presented in the lower extremities, and as the condition advances it becomes generalized. Signs of dyspnea can occur due to having an edema in patients with this condition. Additionally, ascites can develop and it can be evident if the patient is having pleural effusions and diaphragmatic compromises.

A 25-year old woman is admitted to the hospital with swellings in her eyes, legs and feet, and generalized edema. The patient reports fatigue and loss of appetite, increased weight gain, and while she has not observed any blood in her urine, she described it as unusually foamy. Her blood pressure is measured at 110/70 mmHg, pulse rate at 65 bpm, and temperature at 36.6 degrees Celsius. Further blood and urine analysis reveal the following findings: hypoalbuminemia, hyperlipidemia, lipiduria, and proteinuria at 4.0 g/day. Which characterization or condition best describes the patients' clinical manifestations? A. Acute nephritic syndrome B. Nephrotic syndrome C. Asymptomatic hematuria D. Asymptomatic proteinuria

ANS: B Nephrotic syndrome is defined by a collection of clinical findings and manifestations, which include: proteinuria greater than or equal to 3.5 g/day in adults, lipiduria, and an associated hyperlipidemia, and hypoalbuminemia (p. 623). The generalized edema is a defining characteristic of nephrotic syndrome due to the loss of plasma protein (particularly albumin), and consequential decrease of plasma colloidal osmotic pressure, and edema. There is an accompanying salt and water retention which worsens the edema, further manifested as increasing weight, as experienced by the patient. The foamy urine can be a further sign of protein in the urine.

What factors make a patient prone to neurogenic shock? Neurogenic shock can be caused by any factor that inhibits the: a. Parasympathetic nervous system b. Sympathetic nervous system c. Somatic nervous system d. Thalamus

ANS: B Neurogenic shock is caused by any factor that inhibits the sympathetic nervous system.

A nurse takes an adult patient's blood pressure and determines it to be normal. What reading did the nurse obtain? a. Systolic pressure between 140 mm Hg and 150 mm Hg b. Systolic pressure less than 120 mm Hg and diastolic pressure less than 80 mm Hg c. Systolic pressure less than 100 mm Hg regardless of diastolic pressure d. Systolic pressure greater than 140 mm Hg and a diastolic pressure of 100 mm Hg

ANS: B Normal blood pressure has a systolic pressure less than 120 mm Hg and diastolic pressure less than 80 mm Hg. A systolic pressure of 140 mm Hg or more would indicate stage I hypertension. A systolic pressure of less than 100 mm Hg would indicate low blood pressure. A diastolic pressure greater than 90 mm Hg would indicate hypertension.

A 65-year-old male presents for a routine checkup. A blood pressure check reveals a systolic pressure of 160 mm Hg and a diastolic pressure of 70 mm Hg. Which of the following is the most likely cause of this type of pressure elevation? a. Vasospasm b. Rigidity of the aorta c. Decreased sodium intake d. Lung disease

ANS: B Older adults experience stiffening of the arteries, which could lead to hypertension. A vasospasm could lead to chest pain, not hypertension. Increased sodium intake, not decreased, could lead to increased blood volume and hypertension. Lung disease would not lead to hypertension.

A 54-year-old male is diagnosed with peptic ulcer disease. This condition is most likely caused by: a. Hereditary hormonal imbalances with high gastrin levels b. Breaks in the mucosa and presence of corrosive secretions c. Decreased vagal activity and vascular engorgement d. Gastric erosions related to high ammonia levels and bile reflux

ANS: B Peptic ulcer disease is caused by breaks in the mucosa and the presence of corrosive substances. High gastrin occurs, but the disease is due to breaks in the mucosa. Vagal activity increases. Gastric erosions occur, but they are not due to high ammonia.

A 16-year-old male took a recreational drug that altered his level of arousal. Physical exam revealed a negative Babinski sign, equal and reactive pupils, and roving eye movements. Which of the following diagnosis will the nurse most likely see on the chart? a. Psychogenic arousal alteration b. Metabolically induced coma c. Structurally induced coma d. Structural arousal alteration

ANS: B Persons with metabolically induced coma generally retain ocular reflexes even when other signs of brainstem damage are present. Psychogenic arousal activation demonstrates a general psychiatric disorder. Structurally induced coma is manifested by asymmetric responses. Structural arousal alteration does not have drug use as its etiology.

A 54-year-old male complains that he has been vomiting blood. Tests reveal portal hypertension. Which of the following is the most likely cause of his condition? a. Thrombosis in the spleen b. Cirrhosis of the liver c. Left ventricular failure d. Renal stenosis

ANS: B Portal hypertension occurs secondarily to cirrhosis of the liver.

A 64-year-old African-American male is admitted into the ICU with an acute kidney injury due to a serious decline in kidney function and imbalance of electrolyte and fluid levels. Which type of acute kidney injury is a result of obstruction of urine flow to the kidneys? A. Intrarenal injury B. Postrenal injury C. Prerenal injury D. Acute Tubular Necrosis

ANS: B Postrenal injuries are a result from an obstruction of the urine outflow in the kidneys. This obstruction can either occur in the bladder, ureter, or urethra from tumors or calculi. Obstruction in the bladder rarely causes acute renal injury unless a patient has kidney damage or only one kidney. For ureters to produce a renal injury, both ureters must obstructed.

A 25-year-old male died from a gunshot wound to the heart. Upon autopsy, abnormalities in the media of the arterial wall and degenerative changes were detected. Which of the following most likely accompanied this finding? a. Fusiform aneurysm b. Saccular aneurysm c. Arteriovenous malformation d. Thrombotic stroke

ANS: B Saccular aneurysms (berry aneurysms) occur frequently (in approximately 2% of the population) and likely result from congenital abnormalities in the media of the arterial wall and degenerative changes. Fusiform aneurysms (giant aneurysms) occur as a result of diffuse arteriosclerotic changes and are found most commonly in the basilar arteries or terminal portions of the internal carotid arteries. Arteriovenous malformation (AVM) is a tangled mass of dilated blood vessels creating abnormal channels between the arterial and venous systems. Thrombotic stroke would show signs of necrotic tissue, not degenerative changes.

Which principle should the nurse remember while planning care for a patient with spinal shock? Spinal shock is characterized by: a. Loss of voluntary motor function with preservation of reflexes b. Cessation of spinal cord function below the lesion c. Loss of spinal cord function at the level of the lesion only d. Temporary loss of spinal cord function above the lesion

ANS: B Spinal shock is the complete cessation of spinal cord function below the lesion. The reflexes are not preserved in spinal shock.

A serious complication of paraesophageal hiatal hernia is: a. Hemorrhage b. Strangulation c. Peritonitis d. Ascites

ANS: B Strangulation of the hernia is a major complication.

A 52-year-old male presents with pooling of blood in the veins of the lower extremities and edema. The diagnosis is chronic venous insufficiency, and an expected assessment finding of this disorder is: a. Deep vein thrombus formation b. Skin hyperpigmentation c. Gangrene d. Edema above the knee

ANS: B Symptoms include edema of the lower extremities and hyperpigmentation of the skin of the feet and ankles. Edema in these areas may extend to the knees. Symptoms include edema and hyperpigmentation of the skin, but deep vein thrombi do not form. Gangrene does not occur in veins, but in arteries. Edema does not occur above the knee.

A 51-year-old male is admitted to the neurological critical care unit with a severe closed head injury. All four extremities are in rigid extension, his forearm is hyperpronated, and his legs are in plantar extension. How should the nurse chart this condition? a. Decorticate posturing b. Decerebrate posturing c. Caloric posturing d. Excitation posturing

ANS: B The description is of a patient in decerebrate posturing.

A 75-year-old male has severe chest pain and dials 911. Lab tests at the hospital reveal elevated levels of cardiac troponins I and T. Based upon the lab findings, the nurse suspects which of the following has occurred? a. Raynaud disease b. Myocardial infarction (MI) c. Orthostatic hypotension d. Varicose veins

ANS: B The diagnosis of acute MI is made on the basis of serial cardiac biomarker alterations. The cardiac troponins (troponins I and T) are the most specific indicators of MI.

The patient reports generalized muscle weakness. The health care provider orders administration of the medication edrophonium chloride (Tensilon). This medication is used in the diagnosis of: a. Amyotrophic lateral sclerosis b. Myasthenia gravis c. MS d. Autonomic hyperreflexia

ANS: B The diagnosis of myasthenia gravis is made on the basis of a response to edrophonium chloride (Tensilon).

The icteric phase of hepatitis is characterized by which of the following clinical manifestations? a. Fatigue, malaise, vomiting b. Jaundice, dark urine, enlarged liver c. Resolution of jaundice, liver function returns to normal d. Fulminant liver failure, hepatorenal syndrome

ANS: B The icteric phase is manifested by jaundice, dark urine, and clay-colored stools. The liver is enlarged, smooth, and tender, and percussion causes pain; this is the actual phase of illness. Fatigue and vomiting occur during the prodromal stage. Resolution occurs in the recovery phase. Fulminant liver failure does not involve icterus.

A 10-year-old male presents with fever, lymphadenopathy, arthralgia, and nose bleeds. He is diagnosed with rheumatic heart disease. While planning care, which characteristic changes should the nurse remember? a. Blood-borne organisms that adhere to the valvular surface b. Antigens that bind to the valvular lining, triggering an autoimmune response c. High fevers that damage collagen in valve leaflets d. Rheumatoid factor in the blood, stimulating valvular degeneration

ANS: B The immune response cross-reacts with molecularly similar self-antigens in heart, muscle, joints, and the brain, causing an autoimmune response resulting in diffuse, proliferative, and exudative inflammatory lesions in these tissues.

Which of the following lab tests will the nurse check to help diagnose heart failure and provide insight into its severity? a. Renin level b. Brain natriuretic c. Potassium d. Troponin

ANS: B The level of serum brain natriuretic peptide (BNP) can also help make the diagnosis of heart failure and give some insight into its severity. Renin levels would not assist in the diagnosis of heart failure, but are elevated in hypertension. Potassium levels would not assist with the diagnosis of heart failure. Troponin assists in the diagnosis of myocardial infarction.

A 24-year-old male who sustained a head injury and fractured femur develops a stress ulcer. A common clinical manifestation of this ulcer is: a. Bowel obstruction b. Bleeding c. Pulmonary embolism d. Hepatomegaly

ANS: B The most common clinical manifestation is bleeding.

A nurse is teaching a patient about anaphylactic shock. Which information should the nurse include? The onset of anaphylactic shock is usually: a. Mild b. Immediate and life threatening c. Delayed by several hours d. Delayed by 24 hours

ANS: B The onset of anaphylactic shock is usually immediate and life threatening.

A 23-year-old female begins having problems with tiredness, weakness, and visual changes. Her diagnosis is multiple sclerosis (MS). What is occurring in the patient's body? a. Depletion of dopamine in the central nervous system (CNS) b. Demyelination of nerve fibers in the CNS c. The development of neurofibril webs in the CNS d. Reduced amounts of acetylcholine at the neuromuscular junction

ANS: B The pathophysiology of MS includes demyelination of nerve fibers. Depletion of dopamine is related to Parkinson disease. The development of neurofibrils is related to Alzheimer disease. Myasthenia gravis is due to decreased amounts of acetylcholine at the junction.

A 23-year-old female patient reports consistent pain and discomfort on her bladder, persistent urge to urinate, sensation of burning while urinating with passing small amounts of urine, and yellow-pinkish urine is observed for 2 days. The nurse noted there is no signs of pain in the back. The patient received antimicrobial medication (Ciprofloxacin), and the symptoms disappeared after 5 days. Which of the following indicates these symptoms? A. Hematuria B. UTI C. Bladder Cancer D. Ureteral invasion

ANS: B Urinary tract infection, a cystitis, is a most common acute lower UTIs which occur mainly in women and are generally caused by E. coli. Its symptom indicates consistent pain on lower (bladder) urinary tract, persistent urge to urinate, sensation of burning (dysuria), and sometimes blood in urine, hematuria. It is treated with a short course of antimicrobial therapy (ex. Ciprofloxacin). Cranberry pills or juices are one of the suggestions for prevention of frequent UTI.

The primary care provider states that the patient has vasogenic edema. The nurse realizes vasogenic edema is clinically important because: a. It usually has an infectious cause. b. The blood-brain barrier is disrupted. c. Increased intracranial pressure (ICP) is excessively high. d. It always causes herniation.

ANS: B Vasogenic edema is clinically important because the blood-brain barrier (selective permeability of brain capillaries) is disrupted, and plasma proteins leak into the extracellular spaces. Vasogenic does not have an infectious cause. ICP is increased, but not more so than other forms of edema. Vasogenic edema disrupts the blood-brain barrier, but it does not always cause herniation.

A 20-year-old male suffers a severe closed head injury in a motor vehicle accident. He remains in a vegetative state (VS) 1 month after the accident. Which of the following structures is most likely keeping him in a vegetative state? a. Cerebral cortex b. Brainstem c. Spinal cord d. Cerebellum

ANS: B When a person loses cerebral function, the reticular activating system and brainstem can maintain a crude waking state known as a VS. Cognitive cerebral functions, however, cannot occur without a functioning reticular activating system.

Reflux esophagitis is defined as a(n): a. Immune response to gastroesophageal reflux b. Inflammatory response to gastroesophageal reflux c. Congenital anomaly d. Secretory response to hiatal hernia

ANS: B When gastroesophageal reflux leads to an inflammatory response, it is termed reflux esophagitis.

A 65-year-old male with a history of untreated hypertension is now experiencing left heart failure. A nurse recalls his untreated hypertension led to: a. Ventricular dilation and wall thinning b. Myocardial hypertrophy and ventricular remodeling c. Inhibition of renin and aldosterone d. Alterations in alpha and beta receptor function

ANS: B With hypertension comes increased afterload and resistance to ventricular emptying and more workload for the ventricle, which responds with hypertrophy of the myocardium and ventricular remodeling.

A 48-year-old female presents at the ER reporting an acute severe headache, nausea, photophobia, and nuchal rigidity. What does the nurse suspect caused these signs and symptoms? a. Diffuse brain injury b. Subarachnoid hemorrhage c. Epidural hematoma d. Classic concussion

ANS: B With subarachnoid hemorrhage, meningeal irritation occurs, leading to nuchal rigidity.

A 73 year old woman is admitted to the surgical unit prior to a surgical procedure to remove varicose veins. The patient is partially blind and has problems with mobility and manual dexterity. Patient presses the call light when she needs to use the restroom, but the nurse is unable to assist the patient for another 20 minutes. By the time the nurse enters the room, the patient has wet herself. The nurse assess the patient and realizes that the patient suffers from incontinence. What type of incontinence is most likely? A. Overflow Incontinence B. Functional Incontinence C. Urge Incontinence D. Stress incontinence

ANS: B Functional incontinence is when the patient has problems attempting to use the toilet when they feel the need to urinate. Because this patient is partially blind and has problems with mobility and manual dexterity, she is physically unable to get herself to the restroom.

What term should the nurse document for a detached blood clot? a. Thrombus b. Embolus c. Thromboembolus d. Infarction

ANS: C A detached thrombus is a thromboembolus. A thrombus is a clot that remains attached to a vessel wall. An embolus is a bolus of material floating in the blood stream. An infarction is death of tissue.

A 30-year-old female had a seizure that started with her fingers and progressive1y spread up her arm and then extended to her leg, with no loss of consciousness. How should the nurse chart this? a. Myoclonic seizure b. Tonic-clonic seizure c. Focal motor seizure d. Atonic seizure

ANS: C A focal motor seizure involves the extremities and the patient does not experience loss of consciousness. A myoclonic seizure involves symmetrical movements of selected extremities. A tonic-clonic seizure involves all body parts and loss of consciousness. An atonic seizure is without muscle tone.

After being diagnosed with nephrotic syndrome, a patient asks the nurse why his lower extremities are swollen. The nurse's best response to the patient's edema would be: a. The swelling is a result of high amounts of fluid intake b. The swelling is a result of right heart failure c. The swelling is due to a decrease in the plasma colloidal osmotic pressure d. The swelling is due to an increase in the plasma colloidal osmotic pressure

ANS: C A hallmark of nephrotic syndrome is generalized edema that results from a decrease in the plasma colloidal osmotic pressure due to the hypoalbuminemia that develops as albumin is lost from the vascular compartment.

A 40-year-old male develops an intestinal obstruction related to protrusion of the intestine through the inguinal ring. This condition is referred to as: a. Intussusception b. A volvulus c. A hernia d. Adhesions

ANS: C A hernia is a protrusion of the intestine through a weakness in the abdominal muscles or through the inguinal ring. Intussusception is telescoping of one part of the intestine into another; this usually causes strangulation of the blood supply and is more common in infants 10 to15 months of age than in adults. A volvulus is a twisting of the intestine on its mesenteric pedicle, with occlusion of the blood supply, often associated with fibrous adhesions. It occurs most often in middle-aged and elderly men. Adhesions are irritation from surgery or trauma that leads to formation of fibrin and adhesions that attach to intestine, omentum, or peritoneum and can cause obstruction; they are most common in small intestine.

An 8 year-old female patient arrives at the ED complaining of the following: sudden onset of shaking chills, a high fever, unilateral right ache in the loin area of the back, difficult, painful, and frequent urination, presence of pus in the urine, significant malaise, abdominal pain, nausea, and vomiting. The patient has a history of two lower urinary tract infections within the past six months. Pain is reported by the patient upon palpation over the right costovertebral angle. These signs and symptoms are all strong indicators of which of the following conditions? A. Lower Urinary Tract Infection B. Appendicitis C. Acute Pyelonephritis D. Systemic Lupus Erythematosus Glomerulonephritis

ANS: C Acute Pyelonephritis is most commonly seen in children with UTIs. Pyelonephritis is a condition that results from infection of the kidney parenchyma and renal pelvis. This condition involves both a bacterial infection and urine reflux of urine up the ureters and into the kidney. Acute pyelonephritis usually presents itself with an abrupt onset of signs and symptoms that include the following: shaking chills, moderate to high fever, and a constant ache in the loin area of the back that is either unilateral or bilateral, lower urinary tract symptoms (including dysuria, frequency, and urgency), significant malaise, and may also include nausea, vomiting, and abdominal pain. Given this information, it is clear that the patient is most likely suffering from acute pyelonephritis.

Upon assessment of the patient, the nurse finds a widened pulse pressure and throbbing peripheral pulses. Which valve disorder does the nurse suspect? a. Mitral regurgitation b. Mitral stenosis c. Aortic regurgitation d. Aortic stenosis

ANS: C Aortic regurgitation is manifested by widened pulse pressure resulting from increased stroke volume and diastolic backflow. Mitral regurgitation is manifested by heart failure. Mitral stenosis is manifested by pulmonary edema and heart failure. Aortic stenosis is manifested by narrowed pulse pressure.

A 72-year-old female has a history of hypertension and atherosclerosis. An echocardiogram reveals backflow of blood into the left ventricle. Which of the following is the most likely diagnosis documented on the chart? a. Mitral regurgitation b. Mitral stenosis c. Aortic regurgitation d. Aortic stenosis

ANS: C Aortic regurgitation would allow backward flow of blood into the left ventricle.

A 16-year-old female presents with abdominal pain in the right lower quadrant. Physical examination reveals rebound tenderness and a low-grade fever. A possible diagnosis would be: a. Colon cancer b. Pancreatitis c. Appendicitis d. Hepatitis

ANS: C Appendicitis is manifested by right lower quadrant pain with rebound tenderness. Colon cancer may be asymptomatic, followed by bleeding. Pancreatitis is manifested by vomiting. Hepatitis would be manifested by upper abdominal pain, not lower.

A 25-year old white male patient comes in to the Emergency Department at UCLA Medical Center, Santa Monica early in the morning complaining of waking up due to a sharp, severe pain coming from his groin. He explains how he has been going to the bathroom frequently since he has been awake, and also feels a bit nauseous. The ER nurse collects a urine sample from the patient and upon examination of the lab results finds the liquid to have an excess of calcium and oxalate, as well as a low amount of citrate. The pH is 8.5. The nurse determines this man's pain and frequent voiding is due to kidney stones. Based on these clinical manifestations, what type of kidney stones does this man most likely have? A. Uric Acid Stone B. Struvite Stone C. Calcium Stone D. Cystinuric Stone

ANS: C Calcium stones are the most common type of kidney stone to develop and are made up of calcium oxalate or calcium phosphate. Individuals who develop calcium stones present with hypercalciuria, hyperoxaluria, and hypocitraturia. A key clinical manifestation of these stones is renal colic, which is characterized by pain emitting from the groin and ranging from moderate to severe. Stone obstruction in the lower urinary tract can also present with urge incontinence, frequent voiding, and urinary urgency. Additionally, alkaline urine is more likely to cause creation of calcium stones. This patient presents with all the signs and symptoms of having calcium stones. His alkaline urine pH level of 8.5, pain radiating from the groin, frequent voiding, and excess of calcium and oxalate in his urine all indicate and promote the formation of calcium stones

A 60-year-old male is diagnosed with cerebral aneurysm. Where does the nurse suspect the cerebral aneurysm is located? a. Vertebral arteries b. Basilar artery c. Circle of Willis d. Carotid arteries

ANS: C Cerebral aneurysms often occur in the circle of Willis.

A 62-year-old male presents to his primary care provider reporting chest pain at rest and with exertion. He does not have a history of coronary artery disease and reports that the pain often occurs at night. He is most likely experiencing which type of angina? a. Unstable b. Stable c. Prinzmetal d Silent

ANS: C Chest pain that occurs at rest and at night is descriptive of Prinzmetal angina. Unstable angina is a form of acute coronary syndrome that results from reversible myocardial ischemia. Stable angina is predictable and occurs with activity. Silent angina has few, if any, symptoms.

A 20-year-old female is being admitted to the hospital with fever and septic shock. Which set of assessment findings would the nurse expect the patient to exhibit? a. Bradycardia, palpitations, confusion, truncal rash b. Severe respiratory distress, jugular venous distention, chest pain c. Low blood pressure, tachycardia, generalized edema d. Reduced cardiac output, increased systemic vascular resistance, moist cough

ANS: C Clinical manifestations of shock will include a low blood pressure and tachycardia. Tachycardia, not bradycardia, will occur. Severe respiratory distress, jugular vein distention, and chest pain are symptoms of heart failure, particularly pulmonary edema. Cardiac output is reduced, but there is a decrease in systemic vascular resistance.

Complete obstruction of bile flow to the liver would be manifested by: a. Elevated hemoglobin and hematocrit b. Lower-leg edema c. Clay-colored stools d. Hypotension

ANS: C Complete obstruction of bile flow leads to clay-colored stools.

Ms. Lee is a 29 year-old woman come into the doctor's office and complains about her bladder. She says "my bladder always hurts, I feel like I urinate 10 times a day and it also hurts to urinate". The results of her urine tests showed no bacteria in the urine. Which of the following is Ms. Lee experiencing? A. Bladder inflammation B. Cystitis C. Cancer of the bladder D. Pyelonephritis

ANS: C Cystitis or painful bladder syndrome is a conditions that is characterized by urinary frequency, urgency, and severe suprapubic pain. Unlike bladder inflammation which is caused by a bacterial infection.

A 52-year-old male is diagnosed with primary hypertension. He has no other health problems. Present treatment would cause the nurse to anticipate administering which drug to the patient? a. A beta-adrenergic agonist b. An alpha-adrenergic agonist c. A diuretic d. A calcium channel agonist

ANS: C Diuretics have been shown to be the safest and most effective medications for lowering blood pressure and preventing the cardiovascular complications of hypertension.

Acute pancreatitis often manifests with pain to which of the following regions? a. Right lower quadrant b. Right upper quadrant c. Epigastric d. Suprapubic

ANS: C Epigastric or midabdominal pain ranging from mild abdominal discomfort to severe, incapacitating pain is one of the manifestations of pancreatitis.

The most common disorder associated with upper GI bleeding is: a. Diverticulosis b. Hemorrhoids c. Esophageal varices d. Cancer

ANS: C Esophageal varices is the most common disorder associated with upper GI bleeding. Diverticulosis could lead to bleeding, but it would be lower GI rather than upper. Hemorrhoids can lead to bleeding, but they would be upper GI. Duodenal ulcers could lead to upper GI bleeding, but peptic ulcers and varices are identified as more common.

A patient has excessive movement. What disorder will the nurse see documented on the chart? a. Hypokinesia b. Akinesia c. Hyperkinesia d. Dyskinesia

ANS: C Excessive movement is the definition of hyperkinesia. Hypokinesia is decreased movement. Akinesia is loss of movement. Dyskinesia is abnormal movement.

A staff member asks a nurse what foam cells are. What is the nurse's best response? Foam cells in a fatty streak are: a. Deposited adipose cells b. Injured neutrophils c. Macrophages that engulf low-density lipoprotein (LDL) d. Lipid-laden mast cells

ANS: C Foam cells are lipid-laden macrophages that engulf LDL.

A 50-year-old male is experiencing reflux of chyme from the stomach. He is diagnosed with gastroesophageal reflux. This condition is caused by: a. Fibrosis of the lower third of the esophagus b. Sympathetic nerve stimulation c. Loss of muscle tone at the lower esophageal sphincter d. Reverse peristalsis of the stomach

ANS: C Gastroesophageal reflux is due to loss of muscle tone at the lower esophageal sphincter.

A 52-year-old presents with bleeding from the rectum. This condition is referred to as: a. Melena b. Occult bleeding c. Hematochezia d. Hematemesis

ANS: C Hematochezia is bleeding from the rectum. Melena is a black or tarry stool. Occult bleeding is hidden bleeding. Hematemesis is vomiting blood.

A 15-year-old male was struck by a motor vehicle and suffered a traumatic brain injury. Paramedics found him unconscious at the scene of the accident. During the ambulance ride, he regained consciousness and was able to maintain a conversation with the medical staff. Upon arrival to hospital, he was alert and oriented. Physical exam reveals confusion and impaired responsiveness. What is the probable nature of his brain injury based on this history? a. Mild concussion b. Subdural hematoma c. Extradural (epidural) hematoma d. Mild diffuse axonal injury

ANS: C Individuals with extradural hematomas lose consciousness at injury; one-third then become lucid for a few minutes to a few days. Mild concussion is characterized by immediate but transitory confusion that lasts for one to several minutes, possibly with amnesia for events preceding the trauma. Subdural hematomas begin with headache, drowsiness, restlessness or agitation, slowed cognition, and confusion. These symptoms worsen over time and progress to loss of consciousness, respiratory pattern changes, and pupillary dilation. Individuals with mild diffuse axonal injury display decerebrate or decorticate posturing and may experience prolonged periods of stupor or restlessness.

A patient is diagnosed with an upper urinary tract infection with complete obstruction due to a kidney stone. What is NOT a potential clinical manifestation as a result of this diagnosis? A. Urinary Tract Infection B. Dysuria C. Polyuria D. Kidney Failure

ANS: C Individuals with upper urinary tract infection indicate different clinical manifestations depending on the cause and rapidity of development of the obstruction. Polyuria is when an abnormal amount of urine is produced and voided from the body. This would not occur if the patient's urinary tract is completely obstructed because the urine would not be able to escape from the bladder and be voided from the body. Therefore, it is not possible for there to be an abnormal amount of discharge of urine. Polyuria is a possible clinical manifestation of partial obstruction because of an inability to concentrate urine, but not in a complete obstruction of the urinary tract.

Tissue damage in pancreatitis is caused by: a. Insulin toxicity b. Autoimmune destruction of the pancreas c. Leakage of pancreatic enzymes d. Hydrochloric acid reflux into the pancreatic duct

ANS: C Leaked enzymes become activated, initiating autodigestion, inflammation, oxidative stress, and acute pancreatitis.

Which assessment finding by the nurse characterizes a mild concussion? a. A brief loss of consciousness b. Significant behavioral changes c. Retrograde amnesia d. Permanent confusion

ANS: C Mild concussion is characterized by immediate but transitory confusion that lasts for one to several minutes, possibly with amnesia for events preceding the trauma. Individuals with extradural hematomas lose consciousness at injury; one-third then become lucid for a few minutes to a few days. Persons with diffuse brain injury demonstrate behavioral changes. Individuals with a mild concussion experience transient not permanent confusion.

A 60-year-old female was diagnosed with mitral stenosis. As a result, the nurse realizes the patient has incomplete emptying of the: a. Right atrium b. Right ventricle c. Left atrium d. Left ventricle

ANS: C Mitral stenosis would result in incomplete emptying of the left atrium, as the mitral valve is located between the left atrium and left ventricle.

When a patient asks, "What is the cause of multiple sclerosis?" how should the nurse reply? The cause of MS seems to be an interaction between: a. Vascular and metabolic factors b. Bacterial infection and the inflammatory response c. Autoimmunity and genetic susceptibility d. Neurotransmitters and inherited genes

ANS: C Multiple sclerosis is due to an interaction between the autoimmune response and genetics.

Mrs. Smith presents to her primary doctor with lower extremity edema. She says she doesn't know what has caused her to retain water and that she has a history of systemic lupus erythematosus. Upon blood and urine lab results, she presents with 2 g/dL of albuminemia, high LDL, and excess protein in her urine. Because the excess protein is alarming, the doctor decides to monitor her urine output for the rest of the day. By the end of the day, Mrs. Smith has excreted a total of 6.0 g of protein in her urine. What condition does Mrs. Smith have? A. Nephritic syndrome B. Acute kidney failure C. Nephrotic syndrome D. Pyelonephritis

ANS: C Nephrotic syndrome is associated with generalized edema, lipiduria, hypoalbuminemia, and massive proteinuria. Mrs. Smith's lab results point to nephrotic syndrome

An initial assessment finding associated with acute spinal cord injury is _____ the injury. a. Pain below the level of b. Loss of autonomic reflexes above c. Loss of voluntary control below d. Hyperactive spinal reflexes below

ANS: C Normal activity of the spinal cord cells at and below the level of injury ceases because of loss of the continuous tonic discharge from the brain or brainstem and inhibition of suprasegmental impulses immediately after cord injury, thus causing spinal shock. Pain would not be present below the level of the injury because activity of the cells below the level of injury ceases. Autonomic reflexes above the injury remain intact. Activity of the cells below the level of injury cease; thus, hyperactive spinal reflexes below the injury will not occur.

A 30-year-old obese female underwent gastric resection in an attempt to lose weight. Which of the following complications could the surgery cause? a. Constipation b. Acid reflux gastritis c. Anemia d. Hiccups

ANS: C One of the complications is anemia due to iron malabsorption, which may result from decreased acid secretion.

When a nurse checks the patient for orthostatic hypotension, what did the nurse have the patient do? a. Physical exertion b. Eat c. Stand up d. Lie down

ANS: C Orthostatic hypotension refers to a drop in blood pressure when standing up.

A 67-year-old male presents with difficulty urinating. Further discussion shows a previous health history of incontinence, but patient suddenly lost the ability to urinate overall. A physical assessment shows the patients prostate is enlarged, causing an obstruction on the lower urinary tract. According to this information, why did the patient previously suffer from incontinence? A. The patient is also suffering from a urethral stricture, causing incontinence. B. A compensatory mechanism of the bladder is the atrophy of the bladder walls, causing incontinence. C. A compensatory mechanism of the bladder is to become more sensitive to bladder contractions. Causing incontinence. D. A compensatory mechanism of the bladder is to shorten the detrusor muscle contractions, causing incontinence.

ANS: C Over-urination and overall sensitivity is a compensatory mechanism of early urinary retention. As the bladder increases in size, the compensatory mechanisms begin to fail and decompensatory changes begin.

A 70-year-old female is being closely monitored in the neurological critical care unit for a severe closed head injury. After 48 hours, her condition begins to deteriorate. Her pupils are small and sluggish, pulse pressure is widening, and she is bradycardic. These clinical findings are evidence of what stage of intracranial hypertension? a. Stage 1 b. Stage 2 c. Stage 3 d. Stage 4

ANS: C Stage 3 is characterized by decreasing levels of arousal or central neurogenic hyperventilation, widened pulse pressure, bradycardia, and pupils that become small and sluggish. Stage 1 is characterized by an ICP that may not change because of the effective compensatory mechanisms, and there may be few symptoms. Stage 2 is characterized by subtle and transient symptoms, including episodes of confusion, restlessness, drowsiness, and slight pupillary and breathing changes. Stage 4 is characterized by cessation of cerebral blood flow.

A 55-year-old male died of a myocardial infarction. Autopsy would most likely reveal: a. Embolization of plaque from the aorta b. Decreased ventricular diastolic filling time c. Platelet aggregation within the atherosclerotic coronary artery d. Smooth muscle dysplasia in the coronary artery

ANS: C The autopsy would reveal platelet aggregation within an atherosclerotic coronary artery (occlusion of the coronary artery)

The cardinal sign of pyloric stenosis caused by ulceration or tumors is: a. Constipation b. Diarrhea c. Vomiting d. Heartburn

ANS: C The cardinal sign of pyloric stenosis is vomiting.

When a patient with left heart failure starts to have a cough and dyspnea, which principle should the nurse remember? Pulmonary symptoms, common to left heart failure, are a result of: a. Inflammatory pulmonary edema b. Decreased cardiac output c. Pulmonary vascular congestion d. Bronchoconstriction

ANS: C The clinical manifestations of left heart failure are the result of pulmonary vascular congestion and inadequate perfusion of the systemic circulation.

A 20-year-old female suffered from spinal cord injury that resulted from a motor vehicle accident. She had spinal shock lasting 15 days and is now experiencing an uncompensated cardiovascular response to sympathetic stimulation. What does the nurse suspect caused this condition? a. Toxic accumulation of free radicals below the level of the injury b. Pain stimulation above the level of the spinal cord lesion c. A distended bladder or rectum d. An abnormal vagal response

ANS: C The described symptoms indicate autonomic hyperreflexia and are due to a distended bladder or rectum.

Which organ should the nurse monitor closely since it is often the first to fail in patients with multiple organ dysfunction syndrome (MODS)? a. Liver b. Heart c. Lungs d. Pancreas

ANS: C The lungs are the first to fail in MODS.

Which patient is most prone to multiple organ dysfunction syndrome (MODS)? In a patient with: a. Myocardial infarction (MI) b. Pulmonary disease c. Septic shock d. Autoimmune disease

ANS: C The most common cause of MODS is septic shock.

A 55-year-old male died in a motor vehicle accident. Autopsy revealed an enlarged liver caused by fatty infiltration, testicular atrophy, and mild jaundice secondary to cirrhosis. The most likely cause of his condition is: a. Bacterial infection b. Viral infection c. Alcoholism d. Drug overdose

ANS: C The most common cause of cirrhosis is alcoholism.

When a patient asks the nurse what is the most common cause of myocardial ischemia, which statement is the correct response? The most common cause of myocardial ischemia is: a. Idiopathic vasospasm b. Arterial emboli from heart valve c. Atherosclerosis d. Venous emboli

ANS: C The most common cause of myocardial ischemia is atherosclerosis.

A 30-year-old white male recently suffered a cerebrovascular accident. Which of the following is the most likely factor that contributed to his stroke? a. Age b. Gender c. Diabetes d. Race

ANS: C The most likely contributing factor to the patient's stroke is that he has diabetes with a fourfold increase in stroke incidence and an eightfold increase in stroke mortality. Age greater than 65 years is contributing factor. Men are affected, but for the 30-year-old, his type 2 diabetes mellitus contributes to a fourfold increase in stroke incidence and an eightfold increase in stroke mortality. Blacks are affected more than whites, and it is this patient's diabetes that places him at risk.

A 28-year-old female presents with severe chest pain and shortness of breath. She is diagnosed with pulmonary embolism, which most likely originated from the: a. Left ventricle b. Systemic arteries c. Deep veins of the leg d. Superficial veins of the arm

ANS: C The most likely origin of the embolism is from the deep veins of the legs.

Six weeks ago a female patient suffered a T6 spinal cord injury. She then developed a blood pressure of 200/120, a severe headache, blurred vision, and bradycardia. What does the nurse suspect the patient is experiencing? a. Extreme spinal shock b. Acute anxiety c. Autonomic hyperreflexia d. Parasympathetic areflexia

ANS: C The patient is experiencing autonomic hyperreflexia, which is manifested by paroxysmal hypertension (up to 300 mm Hg, systolic), a pounding headache, blurred vision, sweating above the level of the lesion with flushing of the skin, nasal congestion, nausea, piloerection caused by pilomotor spasm, and bradycardia (30 to 40 beats/min).

A 55-year-old female has undiagnosed hypertension. She presents to her primary care provider reporting impaired vision and chronic edema. Lab tests reveal that she also has renal insufficiency. While planning care, the nurse realizes the most likely cause for these findings is: a. Clotting and gangrene b. Free radical injury and cell toxicity c. End-organ damage d. Hypertrophy and hyperplasia

ANS: C The patient is experiencing end-organ damage, as indicated by renal insufficiency.

A 60-year-old female with a recent history of head trauma and a long-term history of hypertension presents to the ER for changes in mental status. MRI reveals that she had a hemorrhagic stroke. What does the nurse suspect caused this type of stroke? a. Rheumatic heart disease b. Thrombi c. Aneurysm d. Hypotension

ANS: C The primary causative factor of a hemorrhagic stroke is an aneurysm.

A 38-year-old female complains of epigastric fullness following a meal, nausea, and epigastric pain. Tests reveal narrowing of the opening between the stomach and the duodenum. This condition is referred to as: a. Ileocecal obstruction b. Hiatal hernia c. Pyloric obstruction d. Hiatal obstruction

ANS: C The pylorus is the opening between the esophagus and the duodenum; the obstruction is pyloric. Ileocecal obstruction is in the small intestine. Hiatal hernia is related to the esophagus. Hiatal obstruction is related to the esophagus.

Which of the following disease processes causes the acute inflammation that occludes the glomerular capillary lumen and deterioration of the capillary wall, which is now only prevalent in underserved populations around the world? A. Before getting infected by Chickenpox, rubella, and smallpox B. Before getting infected by Staphylococcus, Candida Albicans, and viral agents responsible for the mumps, measles, and chickenpox C. After getting infected by Group A B-hemolytic streptococcus, staphylococcus viral agents responsible for the mumps, measles and chickenpox D. After getting infected by Group A B-hemolytic streptococci, tuberculosis and pneumonia

ANS: C This infection is called acute postinfectious glomerulonephritis, that occurs after an initial infection from group A B-hemolytic streptococci, and after other organisms that include staphylococci and viral agents responsible for pumps, measles and chickenpox.

Mrs. Thomason, a 58-year-old women, comes into her local clinic complaining of back pain, vomiting, and a fever. What statement would indicate that Mrs. Thomason has a lower tract infection? A. "I go multiple hours without the need to urinate" B. "My urine is clear and yellow" C. "It burns when urinating" D. "I can't hold the urge to urinate past 3 hours"

ANS: C UTIs cause pain when urinating. Cystitis is a bladder infection and is the most common UTI site. It is the inflammation of the bladder due to a bacterial infection. Pain and difficulty when urinating are common symptoms with bladder infections. Other common signs and symptoms include an increased urge to urinate, presence of blood in urine, abdominal pain, foul odor of urine, or concentrated dark urine. Her fever, vomiting, and back pain are common symptoms related to bladder infections.

A 60-year-old male presents with GI bleeding and abdominal pain. He reports that he takes NSAIDs daily to prevent heart attack. Tests reveal that he has a peptic ulcer. The most likely cause of this disease is: a. Increasing subepithelial bicarbonate production b. Accelerating the H+ (proton) pump in parietal cells c. Inhibiting mucosal prostaglandin synthesis d. Stimulating a shunt of mucosal blood flow

ANS: C Use of NSAIDs inhibit prostaglandins and maintenance of the mucosal barrier and decrease bicarbonate secretion. NSAIDs decrease bicarbonate production. H. pylori increases hydrogen secretion. NSAIDs do not affect mucosal blood flow.

Many valvular stenosis and regurgitation disorders in adults have a common etiology. Which of the following conditions should alert the nurse that the patient may have both types of valve dysfunction? a. Heart failure b. Connective tissue disorders c. Rheumatic fever or heart disease d. Syphilis infection

ANS: C Valvular dysfunction is often related to rheumatic fever.

While planning care, the nurse remembers which principle? In valvular _____, the valve opening is constricted and narrowed because the valve leaflets, or cusps, fail to open completely. a. Regurgitation b. Insufficiency c. Stenosis d. Incompetence

ANS: C Valvular stenosis occurs when the valve opening is constricted and narrowed. Valvular regurgitation occurs when blood moves backward into the chamber from which it came. Valvular insufficiency occurs when blood regurgitates backward into the chamber from where it came. Valvular incompetence leads to regurgitation.

A 45-year-old female presents to the emergency room (ER) reporting excessive vomiting. A CT scan of the brain reveals a mass in the: a. Skull fractures b. Thalamus c. Medulla oblongata d. Frontal lobe

ANS: C Vomiting is due to disruptions in the medulla oblongata.

The most common clinical manifestation of portal hypertension is _____ bleeding. a. Rectal b. Duodenal c. Esophageal d. Intestinal

ANS: C Vomiting of blood from bleeding esophageal varices is the most common clinical manifestation of portal hypertension.

Mrs. Saunders, a 73 year old women comes into the hospital complains of voiding uncontrollably and not being able to prevent it. The nurse runs a urinalysis and blood test on her. During the physical exam, the nurse tells the patient to cough, and the nurse realizes there is an increase in intra-abdominal pressure weakness and the pressure isn't being equally transmitted to the urethra. What would the nurse interpret with these clinical manifestations? A. Urinary tract infection B. Overflow incontinence C. Stress incontinence D. Urge incontinence

ANS: C When a patient comes into the hospital and complains of voiding uncontrollably, it is clear that there is an issue with urinary incontinence. Stress incontinence is a common problem for women of all ages, especially older women. It is the involuntary loss of urine that occurs when the intravesical pressure exceeds the maximum urethral closure pressure which is the pressure difference between the urethra and the bladder.

A 72-year-old male demonstrates left-sided weakness of upper and lower extremities. The symptoms lasted 4 hours and resolved with no evidence of infarction. The patient most likely experienced a(n): a. Stroke in evolution b. Arteriovenous malformation c. Transient ischemic attack d. Cerebral hemorrhage

ANS: C When symptoms resolve with complete recovery, it is a transient ischemic attack. A stroke in evolution is an impending stroke, and symptoms would not resolve. An arteriovenous malformation is an abnormal arrangement of blood vessels that could lead to stroke, but is not a disorder in itself. Cerebral hemorrhage would not resolve.

A 59-year-old female is diagnosed with left ventricular failure. If a decrease in kidney perfusion occurs, the nurse knows this would ultimately cause: a. Decreased left ventricular preload b. Increased pulmonary capillary permeability c. Increased systemic vascular resistance d. Decreased cardiac oxygen demand

ANS: C With a decrease in kidney perfusion, renin is released with the ultimate outcome of increased systemic vascular resistance to raise blood pressure to increase blood flow to the kidney. Ventricular preload will increase, not decrease. Capillary permeability will likely decrease. Cardiac oxygen demand will increase, not decrease.

A 65-year-old male suffers a subarachnoid hemorrhage secondary to uncontrolled hypertension. He appears drowsy and confused with pronounced focal neurologic deficits. This condition is grade: a. I b. II c.III d. IV

ANS: C With grade III, the patient experiences drowsiness and confusion with or without focal neurologic deficits and pronounced meningeal signs. With grade I, neurologic status is intact with mild headache and slight nuchal rigidity. With grade II, neurologic deficit is evidenced by cranial nerve involvement and moderate to severe headache with more pronounced meningeal signs (e.g., photophobia, nuchal rigidity). With grade IV, the patient is stuporous with pronounced neurologic deficits (e.g., hemiparesis, dysphasia) and nuchal rigidity.

Mrs. Meyers has come into the emergency room because she is worried that there is something wrong with her. In the past three weeks, she has noticed that she has been "wetting herself" every time she coughs or sneezes. She also tells you, the nurse, that this does not happen all time. It only occurs when she exerts herself. As the nurse, you are able to diagnose Mrs. Meyers with: A. Urge Incontinence B. Overflow Incontinence C. Functional Incontinence D. Stress Incontinence

ANS: D A person who has stress incontinence will experience a loss of urine involuntarily upon a sudden act of physical exertion. Examples of the physical exertion include lifting, coughing, sneezing, or laughing. The reason for this involuntary loss of urine is that the physical exertion increases the intra-abdominal pressure and this pressure exceeds the maximum urethral closure pressure (the difference between the pressures of the bladder and the urethra). When the intra-abdominal pressure increases, incontinence occurs due to the unequal pressure distribution to the urethra. This is the case for this patient.

A 20-year-old male was at the supermarket when he fell to the ground. Bystanders reported that he lost consciousness and his body tensed up then relaxed, then tensed and relaxed several times. He most likely was experiencing a(n): a. Partial seizure b. Absence seizure c. Myoclonic seizure d. Tonic-clonic seizure

ANS: D A tonic-clonic seizure involves repeated contraction and relaxation. A partial seizure involves only one body part. An absence seizure is without movement. A myoclonic seizure is a simple seizure with minimal jerking present.

A 7-year old named Max was recently treated for strep throat. However, 5 days later, his mom noticed swelling in his hands and face as well as dark colored urine. She brought him into urgent care and collect a urine sample and found elevated levels of protein and blood in the urine. Additionally, they collected a blood sample which came back with lowered levels of C3 and an increased antistreptococcal antibody titer. What condition is Max most likely presenting with? A. Kidney stones B. Pyelonephritis C. Autosomal Dominant Polycystic Kidney Disease D. Glomerulonephritis

ANS: D Acute glomerulonephritis often occurs after an infection from Group A beta-hemolytic Strep. Oliguria is the first initial sign of glomerulonephritis, followed by proteinuria and hematuria. The resulting urine has a dark, soda colored appearance. Because the glomerulus is inflamed, the blood cannot be filtered properly which results in edema, especially in the hands and face

Mr. Scott is currently receiving immunosuppressive therapy. During one of his sessions, he suddenly develops chills and a fever. He also presents with nausea and vomiting. As the nurse assesses him, Mr. Scott states that he feels an ache in the loin area of his back. Upon palpation to the costovertebral angle, his pain intensifies. What is Mr. Scott experiencing? A. Cystitis B. Chronic Kidney Disease C. Chronic pyelonephritis D. Acute pyelonephritis

ANS: D Acute pyelonephritis is an infection of the ureter, renal pelvis, and/or renal parenchyma. It occurs in people who are chronically ill and receiving immunosuppressive therapy. The onset is abrupt and consists of shaking chills, moderate to high fever, and a constant ache in the loin area of the back. Palpation and percussion to the costovertebral angle causes pain and tenderness. Symptoms subside within several days if the patient takes antimicrobial drugs.

When a patient has a peculiar sensation that immediately precedes a seizure, what term should the nurse use to describe this sensation? a. Prodroma b. Agnosia c. Spasm d. Aura

ANS: D An aura is a peculiar sensation that immediately precedes a seizure. A prodroma is a manifestation that occurs hours preceding a seizure. Agnosia is an inability to recognize objects. A spasm is a jerking action.

As a portion of the assessment of the patient with meningitis, the health care provider flexes the patient's neck to the chest. The patient responds with flexion of the legs and thighs. The name for this response is: a. Decorticate posturing b. Decerebrate posturing c. Kernig's sign d. Brudzinski's sign

ANS: D Brudzinski's sign is demonstrated by flexion of the legs and thighs with forceful flexion of the neck onto the chest. Decorticate posturing occurs when the arms are bent forward on the chest and the hands clenched. Decerebrate posturing occurs when the upper extremities extend. Kernig's sign occurs when the patient straightens the knee with the hip and knee in a flexed position.

A 40-year-old male who consumes a diet high in fat and low in fiber is at risk for: a. Cancer of the stomach b. Cancer of the liver and biliary ducts c. Cancer of the small intestine d. CRC

ANS: D CRC is associated with dietary intake, primarily lack of fiber and high fat content.

complaining of painful urination and discomfort that radiates to the lower abdomen and groin. After removing the kidney stone what changes to the patient's diet could be made to decrease his risk of another incident? A. Eat more spinach B. Eat less kale C. Eat more beets D. Eat less nuts and nut butter

ANS: D Calcium-based kidney stones occur when calcium oxalate crystals form and aggregate with organic material in the lumen of the kidney tubule. Nuts and nut butter are high oxalate foods so decreasing intake may reduce the risk

A 50-year-old male suffers a severe head injury when his motorcycle hits a tree. His breathing becomes deep and rapid but with normal pattern. What term should the nurse use for this condition? a. Gasping b. Ataxic breathing c. Apneusis d. Central neurogenic hyperventilation

ANS: D Central neurogenic hyperventilation is a sustained, deep, rapid, but regular, pattern (hyperpnea) of breathing. Gasping is a pattern of deep "all-or-none" breaths accompanied by a slow respiratory rate. Ataxic breathing is completely irregular breathing that occurs with random shallow and deep breaths and irregular pauses. Apneusis is manifested by a prolonged inspiratory pause alternating with an end-expiratory pause.

A 56-year-old male is diagnosed with coronary artery disease. Which of the following modifiable risk factors would the nurse suggest the patient change? a. Eating meat b. Living arrangements c. Drinking tomato juice d. Smoking cigarettes

ANS: D Cigarette smoking leads to vasoconstriction and should be the first behavior the patient changes.

Which of the GI cancers has the highest rate of incidence and is responsible for the highest number of deaths? a. Esophageal b. Stomach c. Pancreatic d. Colorectal

ANS: D Colorectal cancer (CRC) is the third most common cause of cancer and cancer death in the United States for both men and women.

Which condition should cause the nurse to assess for high-output failure in a patient? a. Metabolic alkalosis b. Hypothyroidism c. Hypovolemia d. Anemia

ANS: D Common causes of high-output failure include anemia.

A 75-year-old female has been critically ill with multiple organ dysfunction syndrome (MODS) for longer than a week and has developed a severe oxygen supply and demand imbalance. The statement that best describes this imbalance is which of the following? a. Increased oxygen delivery to cells fails to meet decreased oxygen demands. b. The amount of oxygen consumed by cells depends only on the needs of cells, because there is oxygen in reserve. c. The situation results in supply-independent consumption. d. The reserve has been exhausted, and the amount of oxygen consumed depends on the amount the circulation is able to deliver.

ANS: D In MODS, the reserve has been exhausted and the body cannot meet the oxygenation demands. It is true that oxygen fails to meet demand, but there is no increase in oxygen because reserves are exhausted. There is no oxygen in reserve. The situation is supply and demand, but the demand cannot be met.

The progress notes read: the cerebellar tonsil has shifted through the foramen magnum due to increased pressure within the posterior fossa. The nurse would identify this note as a description of _____ herniation. a. Supratentorial b. Central c. Cingulated gyrus d. Infratentorial

ANS: D In infratentorial herniation, the cerebellar tonsil shifts through the foramen magnum because of increased pressure within the posterior fossa. Supratentorial herniation involves temporal lobe and hippocampal gyrus shifting from the middle fossa to posterior fossa. Central herniation is a type of supratentorial herniation and is the straight downward shift of the diencephalon through the tentorial notch. Gyrus herniation occurs when the cingulate gyrus shifts under the falx cerebri. Little is known about its clinical manifestations.

A 55-year-old female has general symptoms of gallstones but is also jaundiced. IV cholangiography would most likely reveal that the gallstones are obstructing the: a. Intrahepatic bile canaliculi b. Gallbladder c. Cystic duct d. Common bile duct

ANS: D Jaundice is due to obstruction of the common bile duct.

Which of the following symptoms would help a health care provider distinguish between ulcerative colitis and Crohn disease? a. Abdominal pain b. Pattern of remission/exacerbations c. Diarrhea d. Malabsorption

ANS: D Malabsorption is common in Crohn disease and is rare in ulcerative colitis. Both disorders can lead to abdominal pain. Both disorders have a clinical course of remissions and exacerbations. Both disorders lead to diarrhea.

Which statement by a patient indicates teaching was successful regarding myasthenia gravis? Myasthenia gravis results from: a. Viral infection of skeletal muscle b. Atrophy of motor neurons in the spinal cord c. Demyelination of skeletal motor neurons d. Autoimmune injury at the neuromuscular junction

ANS: D Myasthenia gravis is a disorder resulting from autoimmune injury at the neuromuscular junction.

A 75-year-old female suffers a stroke and now has difficulty writing and production of language. This condition is most likely caused by occlusion of the: a. Anterior communicating artery b. Posterior communicating artery c. Circle of Willis d. Middle cerebral artery

ANS: D Occlusion of the left middle cerebral artery leads to the inability to find words and difficulty writing.

A 50-year-old male is diagnosed with orthostatic hypotension. Which of the following symptoms would he most likely experience? a. Headache and blurred vision b. Nausea and vomiting c. Chest pain and palpitations d. Syncope and fainting

ANS: D Orthostatic hypotension is often accompanied by dizziness, blurring or loss of vision, and syncope or fainting.

An older adult is admitted to the ER following a fall. The patient complains of pain in the back. The patient has a history of osteoporosis. The nurse would expect the patient's injury and subsequent pain is most likely due to: a. A fractured hip b. Spinal stenosis c. Herniation of a thoracic disk d. A spinal compression fracture

ANS: D Osteoporosis increases the risk of spinal compression fractures and may be why elderly women report more symptoms than men. A fracture of the hip could have occurred, but the patient's pain is in the back and this would not cause pain in the back. Spinal stenosis is a narrowing of the spinal and is most often congenital and would not be due to a fall. Herniation of a thoracic disk could have occurred, but the most likely explanation is a compression fracture.

A 45-year-old male complains of heartburn after eating and difficulty swallowing. He probably has: a. Pyloric stenosis b. Gastric cancer c. Achalasia d. Hiatal hernia

ANS: D Regurgitation, dysphagia, and substernal discomfort after eating are common in individuals with hiatal hernia. Pyloric stenosis is manifested by projectile vomiting. Gastric cancer is not manifested by heartburn. Achalasia is a form of functional dysphagia caused by loss of esophageal innervation.

Steven Lewis, a 55-year-old previously active Caucasian man, arrives at the medical clinic complaining of pain in his lower abdomen, as well as pain when urinating. His medical history indicates that 2 years ago he was diagnosed with kidney stones, and received treatment. Which of the following is NOT a risk factor for kidney stone development in this patient. A. Gender (Male) B. History of Kidney Stones C. Race (Caucasian) D. Active Lifestyle

ANS: D Risk factors for kidney stones include being male, Caucasian, previous history of kidney stones, living in hot areas, low fluid intake, unhealthy diet, living a sedentary lifestyle, and can occur more frequent during hot seasons. The patient's active lifestyle would not be a risk factor for him having kidney stones.

Betty White is a 96 year-old female patient complaining of involuntary urine leakage. She is aware when it happens, but is unable to prevent it. She says it mostly occurs when she is laughing and this is impeding her ability to do her work as a comedian. Betty is experiencing what type of incontinence? A. Obstruction incontinence B. Old person incontinence C. Urge incontinence D. Stress incontinence

ANS: D Stress incontinence is a common problem for women when the muscles of the pelvic floor become weakened. Because of this, when Betty laughs, her intra-abdominal pressure increases and does not get equally transmitted to the urethra, thus allowing for involuntary urine loss. This is a classic symptom of stress incontinence.

A 50-year-old obese male with hypertension and coronary artery disease visits a nutritionist for food counseling. He has an elevated level of low-density lipoprotein (LDL) and a low level of high-density lipoprotein (HDL). Which of the following should the nurse advise him to avoid? a. Monounsaturated fats b. Polyunsaturated fats c. Saturated fats d. Trans fats

ANS: D Trans fats are primarily found in artificially solidified (hydrogenated) oils (e.g., margarine and vegetable shortening). By becoming more solid, they lose essential fatty acids (EFAs). They can raise LDL and lower HDL levels.

A teenage boy sustains a severe closed head injury following an all-terrain vehicle (ATV) accident. He is in a state of deep sleep that requires vigorous stimulation to elicit eye opening. How should the nurse document this in the chart? a. Confusion b. Coma c. Obtundation d. Stupor

ANS: D Stupor is a condition of deep sleep or unresponsiveness from which the person may be aroused or caused to open eyes only by vigorous and repeated stimulation. Confusion is the loss of the ability to think rapidly and clearly and is characterized by impaired judgment and decision making. Coma is a condition in which there is no verbal response to the external environment or to any stimuli; noxious stimuli such as deep pain or suctioning do not yield motor movement. Obtundation is a mild to moderate reduction in arousal (awakeness) with limited response to the environment.

A 53-year-old male patient is being seen in the hospital with complaints of fatigue and malaise. The patient also notes that he is having trouble sleeping and experiencing significant weight loss. Patient denies having any pain. Blood assays have revealed increased serum creatinine, potassium and urea levels. What is the patient likely experiencing? A. Kidney Stones B. Cystitis C. Pyelonephritis D. CKD

ANS: D Symptoms of Chronic Kidney Disease include fatigue, peripheral neuropathy, restless leg syndrome, sleep disturbance, uremic encephalopathy, anorexia and nausea. The kidneys are responsible for regulating sodium and water balance, excreting potassium, and regulating pH balance in the blood. Therefore, the blood results reveal a marked disturbance in kidney function related to CKD.

A 10-month-old is brought to the pediatrician by the mother who states the baby has been experiencing colicky pain followed by vomiting, sweating, nausea, and irritability. Testing reveals a condition in which one part of the intestine telescopes into another. From which type of intestinal obstruction is he suffering? a. Hernia b. Ileus c. Torsion d. Intussusception

ANS: D Telescoping of one part of the intestine into another; this usually causes strangulation of the blood supply and is more common in infants 10 to 15 months of age than in adults.

A patient comes into the ER complaining about hardly urinating and is seen with ischemic edema of the lower extremities. His serum creatinine, blood urea nitrogen, and serum potassium levels are high and metabolic acidosis has developed. He said this has been going on for a month and it is found that he is suffering from an acute kidney injury. Based on this information, what acute kidney injury phase is this patient going through? A. Recovery B. Diuresis C. Initiation D. Oliguric or Maintenance

ANS: D The maintenance phase of ATN is characterized by a marked decrease in the GFR, causing sudden retention of endogenous metabolites, such as urea, potassium, sulfate, and creatinine, that normally are cleared by the kidneys. The urine output usually is lowest at this point. Fluid retention gives rise to edema, water intoxication, and pulmonary congestion.

A 39-year-old female presents with abdominal pain and jaundice. She is diagnosed with gallstones and undergoes cholecystectomy. An analysis of her gallstones would most likely reveal a high concentration of: a. Phosphate b. Bilirubin c. Urate d. Cholesterol

ANS: D The majority of gallstones are composed of cholesterol.

The most common cause of chronic vascular insufficiency among the elderly is: a. Anemia b. Aneurysm c. Lack of nutrition in gut lumen d. Atherosclerosis

ANS: D The most common cause of chronic vascular insufficiency is atherosclerosis, especially in the elderly.

The majority of intervertebral disk herniations occur between which vertebral levels (cervical, C; thoracic, T; lumbar, L; sacral, S)? a. C1 to C3 b. T1 to T4 c. T12 to L3 d. L4 to S1

ANS: D The most common discs affected by herniation are the lumbosacral disks—that is, L5-S1 and L4-L5.

The nursing student would correctly identify the most common symptom of brain abscess as: a. Nuchal rigidity b. Vomiting c. Drowsiness d. Headache

ANS: D The most common symptom of brain abscess is headache.

A 70-year-old Asian male is admitted to the ICU for heart failure and cardiogenic shock. During his health history interview, the patient states that he drinks eight 8-ounce glasses of water per day and took amphotericin B (antifungal agent). Which of the following is NOT a risk factor for prerenal Acute Kidney Injury and Failure (AKI)? A. Heart Failure and Cardiogenic Shock B. Age C. Amphotericin B D. Fluid Intake

ANS: D The patient drinking eight 8-ounce glasses of water per day provides sufficient fluid intake. It is severe dehydration that is a risk factor for AKI as it depletes vascular volume. Other causes of low vascular volume include hemorrhage and decreased extracellular fluid.

A 60-year-old female had a myocardial infarction. She was brought to the hospital 30 minutes later. She survived, but now the nurse is providing care for impaired ventricular function because: a. There is a temporary alteration in electrolyte balance. b. There is too much stress on the heart. c. The cells become hypertrophic. d. The resulting ischemia leads to hypoxic injury and myocardial cell death.

ANS: D The patient has impaired ventricular functioning because a portion of the myocardium has died due to ischemia.

A 27-year-old male is admitted to a neurologic unit with a complete C-5 spinal cord transection. On initial assessment, he is bradycardic, hypotensive, and hyperventilating. He appears to be going into shock. The most likely mechanism of his shock is: a. Hypovolemia caused by blood loss b. Hypovolemia caused by evaporative fluid losses c. Vasodilation caused by gram-negative bacterial infection d. Vasodilation caused by a decrease in sympathetic stimulation

ANS: D The patient is experiencing neurogenic shock in which blood volume has not changed, but SVR decreases drastically so that the amount of space containing the blood has increased, leading to hypotension. In this type of shock, blood loss has not occurred. In this type of shock, fluid loss has not occurred. Vasodilation due to infection would be septic shock; the type of shock described in the patient is due to loss of sympathetic stimulation.

A 65-year-old female presents to the emergency department reporting difficulty swallowing and shortness of breath. A CT scan would most likely reveal an aneurysm in the: a. Cerebral vessels b. Renal arteries c. Inferior vena cava d. Thoracic aorta

ANS: D Thoracic aortic aneurysms can cause dysphagia (difficulty swallowing) and dyspnea (breathlessness). Aneurysms in cerebral vessels will produce a headache. Aneurysms in the renal arteries will produce flank pain. Aneurysms in the inferior vena cava may produce chest pain.

A 55-year old male is admitted to the hospital and is diagnosed with lower urinary tract obstruction disorder. To relieve bladder distention, the nurse is ordered to insert a urinary catheter. Which of the following is not a compensatory mechanism associated with this disorder? A. The bladder starts to hypertrophy B. Ability to suppress urination is diminished C. Pressure increases due to thickness of the bladder wall D. Bladder overstretches and loses its power of contraction

ANS: D This is the correct answer because this mechanism is a sign of decompensation. When compensatory mechanisms are no longer working, the detrusor muscle contraction period shortens and does not allow the urine to expel fully. The bladder overstretches due to lose of contraction and results in overflow incontinence.

A 15-year-old female presents with breast discharge, dysmenorrhea, and excessive excitability. Tests reveal that all her pituitary hormones are elevated. What does the nurse suspect as the most likely cause for these assessment findings? a. A pituitary adenoma b. Hypothalamic hyposecretion c. Hypothalamic inflammation d. Pheochromocytoma

a. A pituitary adenoma Rationale: Hormonal effects of pituitary adenomas include hypersecretion from the adenoma, itself, and hyposecretion from surrounding pituitary cells; in this case prolactin would be elevated with the manifestation of menstrual irregularities and secretion from the breast.

A nurse is discussing endocrine system dysfunction with a patient. Which statement indicates the patient understood? Endocrine system dysfunction can result from hyposecretion, hypersecretion, or from: a. Abnormal receptor activity b. Abnormal hormone levels c. Increased synthesis of second messengers d. Extracellular electrolyte alterations

a. Abnormal receptor activity Rationale: Dysfunction may result from abnormal cell receptor function or from altered intracellular response to the hormone-receptor complex.

A 29-year-old female presents with cloudy urine, flank pain, hematuria, and fever. Which of the following does the nurse suspect the patient is most likely experiencing? a. Acute cystitis b. Renal calculi c. Chronic renal failure d. Postrenal renal failure

a. Acute cystitis

A 42-year-old female presents with dyspnea; rapid, shallow breathing; inspiratory crackles; decreased lung compliance; and hypoxemia. Tests reveal a fulminant form of respiratory failure characterized by acute lung inflammation and diffuse alveolocapillary injury. Which of the following is the most likely diagnosis the nurse will observe on the chart? a. Acute respiratory distress syndrome (ARDS) b. Sarcoidosis c. Postoperative respiratory failure d. Malignant respiratory failure

a. Acute respiratory distress syndrome (ARDS)

A 60-year-old female with a 25-year history of smoking is diagnosed with emphysema. She has an increased anterior-posterior chest diameter. The nurse attributes this finding to: a. Air trapping b. Decreased inspiratory reserve volumes c. Increased flow rates d. Alveolar destruction

a. Air trapping Rationale: Air trapping expands the thorax, putting the respiratory muscles at a mechanical disadvantage.

A 19-year-old female was involved in a motor vehicle accident during which she sustained a closed head injury. She is now experiencing detrusor sphincter dyssynergia. Which of the following is the most beneficial medication treatment? a. Alpha blocker b. Beta blocker c. Vasodilator d. Diuretic

a. Alpha blocker Rationale: Because the bladder neck consists of circular smooth muscle with adrenergic innervation, detrusor sphincter dyssynergia may be managed by alpha-adrenergic blocking (antimuscarinic) medications.

A nurse is preparing to teach a patient about Addison disease. Which information should the nurse include? The most common cause of Addison disease is: a. An autoimmune reaction b. Dietary deficiency of sodium and potassium c. Cancer d. Viral infection of the pituitary gland

a. An autoimmune reaction Rationale: Addison disease is caused by autoimmune mechanisms that destroy adrenal cortical cells and is more common in women.

A 55-year-old female is admitted to the medical unit for complications of long-term, poorly controlled type 2 DM. Which of the following would the nurse expect to find in addition to elevated glucose? a. Atherosclerosis b. Metabolic alkalosis c. Elevated liver enzymes d. Anemia

a. Atherosclerosis Rationale: Macrovascular disease (lesions in large and medium sized arteries) increases morbidity and mortality and increases risk for accelerated atherosclerosis. Acidosis, rather than alkalosis, would occur in this patient.

A 30-year-old male prison inmate contracted tuberculosis during an outbreak. When the nurse reviews the lab results, the organism that caused this condition is a: a. Bacterium b. Fungus c. Virus d. Parasite

a. Bacterium

A 30-year-old female received a severe head injury in a motor vehicle accident. She is now experiencing respiratory abnormalities characterized by alternating periods of deep and shallow breathing with periods of apnea. What term should the nurse use when charting this condition? a. Cheyne-Stokes b. Frank-Starling c. Apnea d. Orthopnea

a. Cheyne-Stokes Rationale: Cheyne-Stokes respirations are characterized by alternating periods of deep and shallow breathing, with periods of apnea lasting from 15 to 60 seconds. Frank-Starling is related to the stretch of fibers. Apnea is cessation of respirations. Orthopnea is dyspnea that occurs when an individual lies flat.

A 65-year-old female with emphysema presents to the ER for difficulty breathing. Physical exam reveals bluish skin and mucous membranes. How should the nurse chart this condition? Patient has: a. Cyanosis b. Hemoptysis c. Hematemesis d. Ischemia

a. Cyanosis Rationale: Cyanosis is a blue color to the skin. Hemoptysis is the coughing up of blood or bloody secretions. Hematemesis is blood in the vomitus. Ischemia is a lack of blood supply to tissues.

A 30-year-old male presents to his primary care provider reporting visual disturbances. CT reveals a pituitary tumor and lab tests reveal elevated prolactin. He is diagnosed with prolactinoma. Which of the following treatments would the nurse help implement? Administering: a. Dopaminergic agonists b. Calcium c. Insulin d. Radiation

a. Dopaminergic agonists Rationale: Dopaminergic agonists (bromocriptine and cabergoline) are the treatment of choice for prolactinomas. Calcium is used to treat parathyroid disease. Insulin is used to treat diabetes. Radiation is used to treat GH.

A 3-year-old male was diagnosed with congenital hypothyroidism. The parents ask the nurse if left untreated what will happen. What is the nurse's best response? If left untreated, the child would have: a. Mental retardation and stunted growth b. Increased risk of childhood thyroid cancer c. Hyperactivity and attention deficit disorder d. Liver, kidney, and pancreas failure

a. Mental retardation and stunted growth Rationale: Cognitive disability varies with the severity of congenital hypothyroidism and the length of delay before treatment is initiated.

A 60-year-old male is diagnosed with renal failure. While the nurse is reviewing lab results, which of the following lab values would be most consistent with this diagnosis? a. Elevated plasma creatinine level b. Decreased plasma potassium level c. Metabolic alkalosis d. Increased urea clearance

a. Elevated plasma creatinine level Rationale: Creatinine is constantly released from muscle and excreted primarily by glomerular filtration. In chronic kidney disease (CKD), as glomerular filtration rate (GFR) declines, the plasma creatinine level increases by a reciprocal amount to maintain a constant rate of excretion. As GFR continues to decline, plasma creatinine concentration increases. Creatinine is elevated; potassium is also elevated. Metabolic acidosis develops. Decreased urea clearance occurs.

An 80-year-old female develops pneumonia in the hospital. She becomes cyanotic, tachycardic, and develops a fever and cough. Chest x-ray reveals pus in the pleural space. Which of the following is the most likely diagnosis documented on the chart? a. Empyema b. Emphysema c. Pleurisy d. Chyle

a. Empyema Rationale: The presence of microorganisms in the pleural space is termed empyema.

A 12-year-old male is newly diagnosed with type 1 DM. Which of the following tests should the nurse prepare the patient to best confirm the diagnosis? a. Fasting plasma glucose levels b. Random serum glucose levels c. Genetic testing d. Glycosylated hemoglobin measurements

a. Fasting plasma glucose levels Rationale: Fasting blood glucose levels are most beneficial in confirming the diagnosis of diabetes. Random serum levels are not as accurate as fasting. Genetic testing may be important for future determination, but it does not confirm the diagnosis. Glycosylated testing measures glucose control over time.

A 35-year-old female with Graves disease is admitted to a medical-surgical unit. While the nurse is reviewing the lab tests, which results would the nurse expect to find? a. High levels of circulating thyroid-stimulating antibodies b. Ectopic secretion of thyroid-stimulating hormone (TSH) c. Low circulating levels of thyroid hormones d. Increased circulation of iodine

a. High levels of circulating thyroid-stimulating antibodies Rationale: Graves disease results from a form of Type II hypersensitivity in which there is stimulation of the thyroid by autoantibodies directed against the TSH receptor.

A 20-year-old male is in acute pain. An arterial blood gas reveals decreased carbon dioxide (CO2) levels. Which of the following does the nurse suspect is the most likely cause? a. Hyperventilation b. Hypoventilation c. Apnea d. Cyanosis

a. Hyperventilation Rationale: Individuals with hyperventilation blow off CO2. Individuals with hypoventilation retain CO2. Apnea is cessation of breathing. Cyanosis is a blue color to the skin.

A 54-year-old patient with pulmonary tuberculosis (lung infection) is evaluated for syndrome of inappropriate ADH secretion (SIADH). Which of the following electrolyte imbalances would be expected in this patient? a. Hyponatremia b. Hyperkalemia c. Hypernatremia d. Hypokalemia

a. Hyponatremia Rationale: Hyponatremia occurs due to increased water reabsorption by kidneys.

A 57-year-old male presents with cough, sputum production, dyspnea, and decreased lung volume. He is diagnosed with pneumoconiosis. When taking the patient's history, which finding is the most probable cause of his illness? a. Inhalation of silica, asbestos, mica b. Autoimmune disease c. Allergic reactions d. Flail chest

a. Inhalation of silica, asbestos, mica

A 22-year-old female presents with chronic bronchitis. Tests reveal closure of the airway during expiration. While planning care, a nurse recalls this condition is most likely caused by: a. Thick mucus from hypertrophied glands b. Ventilation-perfusion mismatch c. Hyperventilation d. Thinning smooth muscle in the bronchioles

a. Thick mucus from hypertrophied glands Rationale: Chronic bronchitis is defined by hypersecretion of thick mucus.

A 45-year-old male presents with oliguria. He is diagnosed with chronic glomerulonephritis. The nurse knows oliguria is related to: a. Thickening of the glomerular membrane and decreased renal blood flow b. Increased glomerular capillary oncotic pressure and tubular obstruction c. Activation of renin-angiotensin from decreased blood volume d. Vasoconstriction of the efferent arterioles

a. Thickening of the glomerular membrane and decreased renal blood flow Rationale: Changes in the glomerulus are characterized by progressive thickening and fibrosis of the glomerular basement membrane.

While planning care for a patient with renal calculi, the nurse remembers the most important factor in renal calculus formation is: a. urine pH. b. body temperature. c. gender. d. serum mineral concentrations.

a. urine pH.

A 50-year-old diabetic male did not take his medication and is now in metabolic acidosis. He is experiencing Kussmaul respirations. What type of breathing will the nurse observe upon assessment? a. Audible wheezing or stridor b. A slightly increased ventilatory rate, large tidal volumes, and no expiratory pause c. Rapid respirations with periods of apnea d. Very slow inhalations and rapid expirations

b. A slightly increased ventilatory rate, large tidal volumes, and no expiratory pause Rationale: Kussmaul respirations are characterized by a slightly increased ventilatory rate, very large tidal volumes, and no expiratory pause. Audible wheezing is usually associated with conditions such as asthma, and stridor indicates a narrowed airway. Cheyne-Stokes respirations are characterized by alternating periods of deep and shallow breathing, with periods of apnea lasting from 15 to 60 seconds. Kussmaul respirations do not have slow inhalations; bronchiolar disorders have these characteristics.

When the pulmonologist discusses the condition in which a series of alveoli in the left lower lobe receive adequate ventilation but do not have adequate perfusion, which statement indicates the nurse understands this condition? When this occurs in a patient it is called: a. A right-to-left shunt b. Alveolar dead space c. A low ventilation-perfusion ratio d. Pulmonary hypotension

b. Alveolar dead space Rationale: When certain areas of the alveoli experience inadequate perfusion, it is referred to as dead space.

Which of the following shows a correct cause and effect sequence in the development of acute respiratory distress syndrome (ARDS)? a. Impaired alveolar compliance causing decreased surfactant production b. Alveolocapillary membrane injury causing a massive inflammatory response c. Hyaline membrane formation and fibrosis causing pulmonary edema d. Increased alveolocapillary membrane permeability causing metabolic alkalosis

b. Alveolocapillary membrane injury causing a massive inflammatory response Rationale: All disorders causing ARDS cause massive pulmonary inflammation that injures the alveolocapillary membrane and produces severe pulmonary edema and inflammation.

When a nurse observes poststreptococcal glomerulonephritis as a diagnosis on a patient, which principle will the nurse remember? Acute poststreptococcal glomerulonephritis is primarily caused by: a. Swelling of mesangial cells in the Bowman space in response to the presence of bacteria b. Antigen-antibody complex deposition in the glomerular capillaries and inflammatory damage c. Inflammatory factors that stimulate cellular proliferation of epithelial cells d. Accumulation of antiglomerular basement membrane antibodies

b. Antigen-antibody complex deposition in the glomerular capillaries and inflammatory damage

A patient with end-stage renal disease has pruritus. When the patient asks what causes this, what is the nurse's best response? Pruritus, seen in patients with end-stage renal disease, is caused by high levels of: a. Potassium b. Calcium c. Sodium d. Magnesium

b. Calcium

An aide asks the nurse what is the most common cause of elevated levels of antidiuretic hormone (ADH) secretion. How should the nurse respond? a. Autoimmune disease b. Cancer c. Pregnancy d. Heart failure

b. Cancer

A 50-year-old male with a 30-year history of smoking was diagnosed with lung cancer. He was previously exposed to air pollution, asbestos, and radiation at his job. Which of the following should the nurse realize had the greatest impact on the development of his cancer? a. Radiation b. Cigarette smoke c. Asbestos d. Air pollution

b. Cigarette smoke

A nurse recalls asthma is classified by: a. Pathophysiologic differences b. Clinical severity c. Genetic traits d. Treatment outcomes

b. Clinical severity Rationale: The National Asthma Education and Prevention Program offers stepwise guidelines for the diagnosis and management of chronic asthma based on clinical severity.

A 19-year-old female with type 1 DM was admitted to the hospital with altered consciousness and the following lab values: serum glucose 500 mg/dl (high) and serum K+ 2 (low). Her parents state that she has been sick with the "flu" for a week. The diagnosis is hyperosmolar hyperglycemia nonketotic syndrome (HHNKS). What relationship do these values have with her insulin deficiency? a. Increased glucose utilization causes the shift of fluid from the intravascular to the intracellular space. b. Decreased insulin causes hyperglycemia and osmotic diuresis. c. Increased glucose and fatty acid metabolism stimulates renal diuresis and electrolyte loss. d. Increased insulin use results in protein catabolism, tissue wasting, and electrolyte loss.

b. Decreased insulin causes hyperglycemia and osmotic diuresis. Rationale: Because the amount of insulin required to inhibit fat breakdown is less than that needed for effective glucose transport, insulin levels are sufficient to prevent excessive lipolysis and ketosis. Volume is depleted, not increased. Electrolyte loss does occur, but it is not due to fatty acids and glucose metabolism, it is due to insufficient insulin. Insulin is decreased, not increased.

A 22-year-old male is admitted to the intensive care unit with a closed head injury sustained in a motorcycle accident. The injury has caused severe damage to the posterior pituitary. Which of the following complications should the nurse anticipate? a. Dilutional hyponatremia b. Dehydration from polyuria c. Cardiac arrest from hyperkalemia d. Metabolic acidosis

b. Dehydration from polyuria Rationale: Diabetes insipidus is a well-recognized complication of closed head injury and is manifested by polyuria leading to dehydration.

A 35-year-old male received a traumatic brain injury in a motor vehicle accident. CT scan revealed a lesion above the pontine micturition center. Which of the following would the nurse expect? a. Dyssynergia b. Detrusor hyperreflexia c. Detrusor areflexia d. Detrusor sphincter dyssynergia

b. Detrusor hyperreflexia Rationale: Neurologic disorders that develop above the pontine micturition center result in detrusor hyperreflexia, also known as an uninhibited or reflex bladder. Lesions that develop in upper motor neurons of the brain and spinal cord result in dyssynergia. Lesions that involve the sacral micturition center (below S1; may also be termed cauda equina syndrome) or peripheral nerve lesions result in detrusor areflexia (acontractile detrusor), a lower motor neuron disorder. Neurologic lesions that occur below the pontine micturition center but above the sacral micturition center (between C2 and S1) are also upper motor neuron lesions and result in detrusor hyperreflexia with vescico-sphincter dyssynergia

A 10-year-old female develops pneumonia. Physical exam reveals subcostal and intercostal retractions. She reports that breathing is difficult and she feels she cannot get enough air. What term should the nurse use to document this condition? a. Cyanosis b. Dyspnea c. Hyperpnea d. Orthopnea

b. Dyspnea Rationale: Dyspnea is defined as "a subjective experience of breathing discomfort that consists of qualitatively distinct sensations that vary in intensity." Cyanosis is a bluish discoloration to the skin. Hyperpnea is an increased ventilatory rate. Orthopnea is dyspnea that occurs when an individual lies flat.

A 30-year-old male is involved in a motor vehicle accident and sustains trauma to the lungs and chest wall. He experiences respiratory failure. Which of the following lab values would the nurse expect? a. Electrolyte imbalances b. Elevated PaCO2 c. Low hematocrit d. Elevated pH

b. Elevated PaCO2 Rationale: In respiratory failure, inadequate gas exchange occurs such that PaO2 = 50 mm Hg or PaCO2 = 50 mm Hg with pH = 7.25. Electrolyte imbalances do not occur, but changes in blood gas values do. Hematocrit may be unaffected. pH will be decreased.

A 55-year-old male presents reporting urinary retention. Tests reveal that he has a lower urinary tract obstruction. Which of the following is of most concern to the nurse? a. Vesicoureteral reflux and pyelonephritis b. Formation of renal calculi c. Glomerulonephritis d. Increased bladder compliance

b. Formation of renal calculi Rationale: Urine stasis occurs with urinary tract obstruction and can lead to the formation of renal calculi.

While checking the lab results for a patient with Graves disease, the nurse would check the T3 level to be abnormally: a. Low b. High c. Variable d. Absent

b. High

When a staff member asks the nurse what causes the chronic complications of DM such as microvascular and macrovascular disease, how should the nurse respond? These complications are primarily related to: a. Pancreatic changes b. Hyperglycemia c. Ketone toxicity d. Hyperinsulinemia

b. Hyperglycemia

When a patient asks what causes hyperglycemia in type 2 DM, how should the nurse respond? Hyperglycemia is a result of: a. Insulin deficiency b. Hyperinsulinemia c. Glucagon deficiency d. Liver dysfunction

b. Hyperinsulinemia Rationale: Type 2 diabetes is due to hyperinsulinemia and insulin resistance.

A 45-year-old female presents with hypertension, anorexia, nausea and vomiting, and anemia. She is diagnosed with chronic renal failure. When the patient asks what caused this anemia, how should the nurse respond? Your anemia is caused by: a. Red blood cells being lost in the urine b. Inadequate production of erythropoietin c. Inadequate iron absorption in the gut d. Red blood cells being injured as they pass through the glomerulus

b. Inadequate production of erythropoietin

A 35-year-old female was severely burned and is hospitalized. She is now suffering from acute tubular necrosis (ATN). Which of the following is the most likely diagnosis the nurse will observe on the chart? a. Prerenal b. Intrarenal c. Extrarenal d. Postrenal

b. Intrarenal Rationale: Intrarenal acute kidney failure is associated with several systemic diseases but is commonly related to ATN. Prerenal renal failure occurs anterior to the kidney. Extrarenal renal failure occurs outside the kidney. Postrenal is due to diseases that obstruct the flow of urine from the kidneys.

A 30-year-old female with Graves disease is admitted to a hospital unit for the surgical removal of her thyroid gland. During the postoperative period, the nurse notes that the patient's serum calcium is low. The nurse should observe the patient for which of the following signs/symptoms? a. Muscle weakness and constipation b. Laryngeal spasms and hyperreflexia c. Abdominal pain and fever d. Anorexia, nausea, and vomiting

b. Laryngeal spasms and hyperreflexia Rationale: Symptoms of low calcium are associated with tetany, a condition characterized by muscle spasms, hyperreflexia, clonic-tonic convulsions, and laryngeal spasms.

A nurse is teaching staff about pulmonary edema. Which information should the nurse include? The most common cause of pulmonary edema is: a. Right heart failure b. Left heart failure c. Asthma d. Lung cancer

b. Left heart failure

A 53-year-old male with a 20-year history of smoking is diagnosed with emphysema. When a staff member asks why the patient's airways are obstructed, how should the nurse respond? The airways are obstructed because of: a. Excessive mucus production b. Loss of elastic recoil c. Infection and inflammation d. Airway edema

b. Loss of elastic recoil

While planning care for a patient with hypothyroidism, which principle should the nurse remember? The basal metabolic rate is unusually _____ with hypothyroidism. a. High b. Low c. Steady d. Variable

b. Low

A nurse is reviewing lab results for glycosylated hemoglobin (hemoglobin A1c) levels. A nurse recalls the purpose of this test is to: a. Measure fasting glucose levels. b. Monitor long-term serum glucose control. c. Detect acute complications of diabetes. d. Check for hyperlipidemia.

b. Monitor long-term serum glucose control. Rationale: Glycosylated hemoglobin refers to the permanent attachment of glucose to hemoglobin molecules and reflects the average plasma glucose exposure over the life of a red blood cell (approximately 120 days). Glycosylated hemoglobin does not measure fasting, but glucose control over time. Glycosylated hemoglobin does not identify complications, but could provide data if the patient is at risk. Glycosylated does not check for hyperlipidemia.

While planning care for a patient from general anesthesia, which principle should the nurse remember? A side effect of some general anesthetic agents is _____ diabetes insipidus. a. Neurogenic b. Nephrogenic c. Psychogenic d. Allogenic

b. Nephrogenic

A 60-year-old male with a 40-year history of smoking presents with chest pain, cough, sputum production, and pneumonia. Tests reveal widespread metastatic cancer, and the primary care provider plans radiation therapy. Which of following is the most likely type of cancer to be documented on the chart? a. Adenoma b. Non-small cell carcinoma c. Small cell carcinoma d. Basal cell carcinoma

b. Non-small cell carcinoma Rationale: For individuals with non-small cell carcinoma, adjunctive radiation and chemotherapy may improve outcomes.

A 60-year-old female with emphysema is having difficulty expiring a given volume of air. When giving report, the nurse will relay that the patient is most likely experiencing _____ pulmonary disease. a. Restrictive b. Obstructive c. Atelectatic d. Pleuritic

b. Obstructive Rationale: Emphysema is a form of obstructive pulmonary disease.

Visual disturbances are a common occurrence in patients with untreated Graves disease. The endocrinologist explains to the patient that the main cause of these complications is: a. Decreased blood flow to the eye b. Orbital edema and protrusion of the eyeball c. TSH neurotoxicity to retinal cells d. Local lactic acidosis

b. Orbital edema and protrusion of the eyeball Rationale: Visual disturbances with Graves disease include orbital fat accumulation, inflammation, and edema of the orbital contents resulting in exophthalmos (protrusion of the eyeball), periorbital edema, and extraocular muscle weakness leading to diplopia (double vision).

A 25-year-old female presents with burning urination. She was diagnosed with a urinary tract infection. When the nurse checks the culture results, which of the following organisms is most likely infecting her urinary tract? a. Streptococcus b. Candida albicans c. Chlamydia d. Escherichia coli

d. Escherichia coli

A 25-year-old female with Graves disease is admitted to a medical-surgical unit. Palpation of her neck would most likely reveal: a. A normal-sized thyroid b. A small discrete thyroid nodule c. Multiple discrete thyroid nodules d. Diffuse thyroid enlargement

d. Diffuse thyroid enlargement Rationale: A patient with Graves disease would reveal stimulation of the gland causing diffuse thyroid enlargement. In Graves disease, the thyroid will be enlarged, not normal sized.

Which of the following patients is at highest risk for developing pulmonary embolism (PE)? a. 21-year-old male with a hemophilia bleeding disorder b. 28-year-old woman who had a baby 6 months earlier c. 36-year-old woman with a history of alcohol abuse who is recovering from a gastric ulcer d. 72-year-old male who is recovering from hip replacement surgery in the hospital

d. 72-year-old male who is recovering from hip replacement surgery in the hospital Rationale: The 72-year-old is at risk for immobility and at increased risk for PE.

The body's inability to conserve water and sodium when affected by Addison disease is explained by which of the following conditions? a. Elevated levels of cortisol b. Decreased levels of ACTH c. Hypersecretion of ADH d. Aldosterone deficiency

d. Aldosterone deficiency Rationale: The symptoms of Addison disease are primarily a result of hypocortisolism and hypoaldosteronism.

A 70-year-old female is in the hospital for pelvic fracture. She develops pulmonary thromboembolism. The nurse realizes this embolus is composed of: a. Fat b. Air c. Tissue fragment d. Blood clot

d. Blood clot

A 45-year-old male undergoes lung transplantation. He now suffers from airway occlusion secondary to fibrosis. Which diagnosis will the nurse see on the chart? a. Compression atelectasis b. Bronchiectasis c. Bronchiolitis d. Bronchiolitis obliterans

d. Bronchiolitis obliterans Rationale: Bronchiolitis is a serious complication of stem cell and lung transplantation and can progress to bronchiolitis obliterans, a fibrotic process that occludes airways and causes permanent scarring of the lungs.

A 44-year-old patient with pulmonary tuberculosis (lung infection) is evaluated for SIADH. Which of the following assessment findings would be expected in this patient? a. Peripheral edema b. Tachycardia c. Low blood pressure d. Concentrated urine

d. Concentrated urine Rationale: Clinical manifestations of SIADH include urine that is inappropriately concentrated with respect to serum osmolarity.

A 15-year-old female is diagnosed with restrictive lung disease caused by fibrosis. The patient had a pulmonary functions test. Which of the following findings is expected? a. Increased compliance b. Increased tidal volume c. Decreased respiratory rate d. Decreased functional residual capacity

d. Decreased functional residual capacity Rationale: Fibrosis progressively obliterates the alveoli, respiratory bronchioles, and interstitium (fibrosing alveolitis), which can result in chronic pulmonary insufficiency, and functional residual capacity declines.

A 19-year-old female with type 1 DM was admitted to the hospital with the following lab values: serum glucose 500 mg/dl (high), urine glucose and ketones 4+ (high), and arterial pH 7.20 (low). Her parents state that she has been sick with the "flu" for a week. Which of the following statements best explains her acidotic state? a. Increased insulin levels promote protein breakdown and ketone formation. b. Her uncontrolled diabetes has led to renal failure. c. Low serum insulin promotes lipid storage and a corresponding release of ketones. d. Insulin deficiency promotes lipid metabolism and ketone formation.

d. Insulin deficiency promotes lipid metabolism and ketone formation. Rationale: With insulin deficiency, lipolysis is enhanced, and there is an increase in the amount of nonesterified fatty acids delivered to the liver. The consequence is increased glyconeogenesis contributing to hyperglycemia and production of ketone bodies (acetoacetate, hydroxybutyrate, and acetone) by the mitochondria of the liver at a rate that exceeds peripheral use.

When the nurse is asked what causes asthma, how should the nurse respond? Asthma is thought to be caused by: a. An autosomal recessive trait b. Autoimmunity c. Excessive use of antibiotics as a young child d. Interactions between genetic and environmental factors

d. Interactions between genetic and environmental factors

A 49-year-old female is diagnosed with hypercortisolism. Which of the following would the nurse expect? a. Weight loss b. Hypoglycemia c. Decreased urination d. Osteoporosis

d. Osteoporosis Rationale: The effects of hypercortisolism in bone cause loss of the protein matrix leading to osteoporosis, with pathologic fractures, vertebral compression fractures, bone and back pain, kyphosis, and reduced height.

What problem should the nurse assess for in a patient with chronic hyperparathyroidism? a. Seizure disorders b. Vitamin D malabsorption c. Hyponatremia d. Osteoporosis and pathologic fractures

d. Osteoporosis and pathologic fractures Rationale: Excessive osteoclastic and osteocytic activity resulting in bone resorption may cause pathologic fractures, kyphosis of the dorsal spine, and compression fractures of the vertebral bodies.

A 30-year-old female presents with hypertension, headache, tachycardia, impaired glucose tolerance, and weight loss. Which of the following diagnosis will the nurse see documented on the chart? a. Addison disease b. Conn disease c. Cushing disease d. Pheochromocytoma

d. Pheochromocytoma


Ensembles d'études connexes

MENINGES AND CEREBROSPINAL FLUID

View Set

ACE GFI PRACTICE (CHAPTER 4 TO 7)

View Set

Chapter 5 ElasticityElasticity is a measure of responsiveness (sensitivity) of the quantity demanded of a good to a change in some variable

View Set

KOHAKÄÄNE EESSÕNAGA -НА 6. klass

View Set